Constitutional BAR

¡Supera tus tareas y exámenes ahora con Quizwiz!

o State modifications of contracts will be permissible if the modifications 3

(1) serve an important and legitimate public interest and (2) are necessary to achieve that public interest; and if (3) the contract impairment is reasonable under the circumstances. o

when is a case ripe?

A case is "unripe" if it doesn't present an immediate threat of harm.

what is the scrutiny used for Gender?

1. Intermediate scrutiny. o Test: a law must be substantially related to an important state interest in order to be valid. § The government must show it is: 1. Substantially related 2. To an important government interest

Do we have equal protection for federal law?

no 1. because the Equal Protection Clause, as part of the Fourteenth Amendment, does not apply to the federal government. (But equal protection principles are made applicable to the federal government through the Fifth Amendment's Due Process Clause — anything that would violate the Equal Protection Clause if done by a state government would violate the Fifth Amendment Due Process Clause if done by the federal government.)

analysis of commerce clause If no Federal law?

ask whether state regulation unduly burdens interstate commerce

Congress enacted the Health Care Act (HCA) "to ensure all Americans access to health care at a reasonable cost." Congress delegated to an executive agency, the Department of Health & Human Services (HHS), responsibility for promulgating regulations to implement the HCA. The HCA further provided that a joint House/Senate committee can repeal or revise the HHS regulations if the committee determines that they inadequately fulfill the HCA's purpose. Would a court be likely to hold that the HCA is unconstitutional? A Yes, because it delegates legislative power to an executive agency. B Yes, because it contains a legislative veto provision. C No, because Congress is reasonably trying to vindicate its Article I legislative power by ensuring the accountability of executive agencies that make law. D No, because the joint committee action to repeal or revise an HHS regulation would not constitute the exercise of executive power.

Answer choice B is correct. Congress cannot authorize a legislative committee to "veto" agency regulations. The Constitution requires that legislative power be exercised in accord with the bicameralism (i.e., passed by both Houses of Congress) and Presentment Clauses of Article I (i.e., the requirement that, in order for a bill to become law it must be presented to the president for approval or return). Answer choice A is incorrect because Congress is not prohibited from delegating legislative power to an executive agency so long as Congress specifies intelligible standards for the agency to follow. Answer choice C is incorrect because, although Congress is reasonably trying to vindicate its Article I legislative power by ensuring the accountability of executive agencies that make law, the chosen means constitute an unconstitutional legislative veto. Answer choice D is incorrect because, although the joint committee action would constitute the exercise of legislative rather than executive power, the action would not comply with the constitutional requirements.

what is the scrutiny used for classifications based on poverty/wealth?

Rational Basis § Must be: 1. Rationally related 2. To a legitimate government interest

Content-based regulation is generally only found constitutional when the speech to be regulated falls into one of the following categories:

obscenity, subversive speech, fighting words, defamation, or commercial speech.

when is a case moot?

· A case is moot if it's been resolved or rendered academic before it reaches the court. exceptions to the "moot" rule: If an event is recurring but will always evade review, the court will address the issue anyway (otherwise, the issue may never be resolved). Such as pregnant women questioning abortion

A professional football association implemented a policy requiring football teams that were part of the association to interview black candidates for any open high-level coaching positions, although no quota was established or preference given to black applicants. The purpose of the policy was to attempt to remedy a history of discrimination in the association wherein black applicants were less likely to be hired and more likely to be fired than their white counterparts. As a result of this policy, more black coaches were hired. Due to the success of the policy, and because other professional sports teams also had a history of discrimination in hiring black coaches, Congress enacted a statute requiring all professional sports teams to implement a similar policy. A professional baseball team has challenged the constitutionality of the statute as an equal protection violation.Is the statute constitutional? A No, because the statute is attempting to remedy general societal discrimination. B No, because the statute is not rationally related to a legitimate government interest. C Yes, because remedial race-based classifications further an important government interest. D Yes, because the statute is remedying specific past discrimination in the hiring of black coaches.

Answer choice A is correct. Although there is no federal equal protection clause, the Supreme Court has held that the Fifth Amendment Due Process Clause includes the rights guaranteed by the Equal Protection Clause, thereby making discrimination by the federal government subject to review under the same standards as discrimination by the states. The Equal Protection Clause provides that states are generally prohibited from passing legislation that treats similarly situated persons differently. Programs that favor racial or ethnic minorities are subject to strict scrutiny. For a governmental affirmative action program based on race to survive, the relevant governmental entity (here, Congress) must show more than a history of societal discrimination. The government must itself be guilty of specific past discrimination against the group it is seeking to favor, and the remedy must be narrowly tailored to end that discrimination and eliminate its effects. In other words, the elimination of past discrimination in a particular governmental institution is a compelling state interest; attempting to remedy general societal injustice through affirmative action is not. Here, there is no evidence presented to show that a particular governmental institution is guilty of specific past discrimination. Rather, it is the professional sports teams that have engaged in past discrimination. The statute enacted by Congress is thus attempting to remedy a general social injustice through affirmative action, instead of specific governmental conduct. Therefore, the statute is unconstitutional. Answer choice B is incorrect. Programs that favor racial or ethnic minorities are subject to strict scrutiny, not rational basis analysis as set forth in this answer choice. Moreover, the statute would likely satisfy the rational basis test as laws are presumed valid under the test. Answer choice C is incorrect because any race-based classifications are subject to strict scrutiny, and this answer choice applies the intermediate scrutiny test. Answer choice D is incorrect, because, as stated above, a specific governmental institution is not guilty of not hiring black applicants to fill high-level coaching positions. LEARN WHYNEXT QUESTION

The director of a community outreach program operated by a local government agency uncovered information that an employee of the program who was also a public official was grossly over reporting the number of hours she worked on her time sheets. The director fired the employee. Consequently, the employee became the subject of a criminal investigation that lead to her being charged with fraud and theft. In response to subpoenas, the director, who himself was an at-will employee, truthfully testified under oath both before a grand jury and at trial about the dismissed employee's falsified time sheets. Shortly thereafter, the director was fired in retaliation for his testimony.Can the director successfully seek reinstatement? A Yes, because the director was fired in retaliation for speech made as a citizen about a matter of public concern. B Yes, because the director testified pursuant to subpoenas. C No, because the director was speaking as a public employee pursuant to his official duties about matters learned in the course of his employment. D No, because the director was an at-will employee who may be dismissed without cause.

Answer choice A is correct. When a government employee contends that his rights under the Free Speech Clause of the First Amendment have been violated by his employer, the employee must show that he was speaking as a citizen on a matter of public concern. In determining whether a government employee is speaking pursuant to his official duties, the critical question is whether the speech at issue is itself ordinarily within the scope of an employee's duties, not whether it merely concerns those duties. When a public employee is speaking about a matter of public concern as a citizen rather than as employee, the First Amendment interest of the employee must be balanced against the interest of the state, as an employer, in effective and efficient management of its internal affairs. Here, the director was compelled to testify and did so truthfully. There is no evidence in the fact pattern to suggest that testifying in this manner was ordinarily within the scope of his duties. Rather, the testimony merely concerned information he learned in the course of his duties. Moreover, the fact pattern explicitly provides that his firing was done in retaliation for that testimony rather than on the basis of the government's needs as an employer. Consequently, the director can successfully seek reinstatement. Answer choice B is incorrect. Although the director was compelled to testify, he still could have been dismissed if his official duties typically involved testifying for the community outreach program. Here, the reason that the director may seek reinstatement is not because he was subpoenaed, but because he was fired in retaliation for protected speech as a citizen on a matter of public concern. Answer choice C is incorrect. In determining whether a government employee is speaking pursuant to her official duties, the critical question is whether the speech at issue is itself ordinarily within the scope of an employee's duties, not whether it merely concerns those duties. In this case, although the director was testifying about matters that he learned during the course of his employment, he was testifying in response to subpoenas as a citizen rather than as a public employee. Answer choice D is incorrect. Although an at-will public employee may not have a legitimate property interest in continued public employment and may generally be dismissed without cause, even an at-will employee may not be dismissed for reasons that in and of themselves violate the Constitution, such as in retaliation for protected speech.

A federal statute was enacted that banned all individuals born on a specific tiny sovereign island nation from entering the United States after it was established that only such individuals were carriers of a highly contagious and deadly virus. A U.S. citizen who had been born on the island nation has challenged the constitutionality of this statute in federal court after being denied reentry to the United States.Which of the following constitutional clauses provides the best grounds for this challenge? A The Comity Clause of Article IV. B The Due Process Clause of the Fifth Amendment. C The Naturalization Clause of Article I, Section 8. D The Privileges or Immunities Clause of the Fourteenth Amendment.

Answer choice B is correct. Although there is no federal equal protection clause, the Supreme Court has held that the Due Process Clause of the Fifth Amendment, which applies to the federal government, incorporates the Equal Protection Clause of the Fourteenth Amendment, which applies to the states. The Equal Protection Clause has been held to protect persons against discrimination on the basis of national origin, as well as race and ethnicity. In this case, because such discrimination under the Equal Protection Clause is subject to strict scrutiny, this argument represents the plaintiff's best argument. (Note: Congress has plenary power over aliens under Article I—a federal alienage classification is likely valid unless it is arbitrary and unreasonable. Although the statute might withstand a challenge by a person born on the island who remains a citizen of that island nation, the plaintiff in this case is a U.S. citizen, and is more likely to succeed.) Answer choice A is incorrect because the Comity Clause (also known as the Privileges and Immunities Clause) of Article IV prohibits a state from discriminating against the citizens of another state. It does not apply to the federal government. Answer choice C is incorrect because the Naturalization Clause of Article I, Section 8, has been interpreted as a source of Congressional power over immigration and aliens. If anything, it would provide support for the constitutionality of this statute rather than support for a challenge to its constitutionality. Answer choice D is incorrect because the Privileges or Immunities Cause of the Fourteenth Amendment prohibits state action that denies natural persons of the privileges or immunities of national citizenship. It has not been applied to actions taken by the federal government.

A state statute prohibits any type of protesting on the public sidewalks immediately in front of an abortion clinic for one hour after the clinic opens, and for one hour prior to the clinic closing. The purpose of the statute was to make sure that the clinic's staff could safely arrive for their workdays and leave at the end of the day, ensuring that the clinics could consistently and safely operate with a full staff. A man's wife recently died from a fatal infection caused by negligence during a procedure at an improperly staffed abortion clinic. One morning, the man sat on the sidewalk in front of an abortion clinic from the moment it opened until the time it closed, crying silently and holding a huge sign reading "Do your jobs!" The man was prosecuted for violating the state statute. The man has claimed that the statute is unconstitutional, and in the alternative, that it was unconstitutionally applied to him.Is the man likely to succeed? A No, because the statute was necessary for the state to achieve a compelling governmental interest. B No, because the statute was content-neutral and narrowly tailored to serve an important government interest. C Yes, because the statute was vague and did not delineate the type of speech prohibited. D Yes, because prosecuting the man does not directly advance the government's asserted interest.

Answer choice B is correct. Applicable to the states via the Fourteenth Amendment, the First Amendment generally prohibits the government's ability to restrict speech. However, the government may impose reasonable restrictions on the time, place, and manner of protected speech in a public forum, such as a public sidewalk, provided the restrictions are (i) content neutral as to both subject matter and viewpoint, (ii) narrowly tailored to serve a significant government interest, and (iii) leave open alternate channels for communication of the information. Here, the statute is content neutral because it does not prohibit only certain types of speech or certain viewpoints—it prohibits all protesting. It is narrowly tailored because it only prohibits speech during hours when protests might prevent clinic staff from entering or leaving. Finally, the statute leaves open alternate channels for communication of the information, as it only restricts protesting for two hours during the day. Thus, the statute is likely constitutional. The statute also does not grant law enforcement unfettered discretion in its application. Additionally, it has not been applied in a manner suggesting a content-based intent. Therefore, there is no evidence that the statute has been unconstitutionally applied to the man. Answer choice A is incorrect because it incorrectly applies the rule for a content-based regulation, and as stated above, the regulation here was content-neutral. Answer choice C is incorrect. A statute is void for vagueness if it fails to provide a person of ordinary intelligence with fair notice of what is prohibited. Here, the statute is not vague, as it gives fair notice of the time, place, and manner in which all protesting was prohibited. Answer choice D is incorrect because the application of this constitutional content-neutral statute to the man is not unconstitutional merely because his speech encouraged the clinic staff to "do their jobs." This regulation does not grant law enforcement unfettered discretion in its application, and has not been applied in a manner suggesting a content-based intent. Therefore, the man has no grounds to assert that the statute is unconstitutional as applied to him.

In order to improve the safety of passenger train services on northeast corridor routes, Congress imposed a $25 tax on all tickets sold by passenger train services for travel throughout the United States.As an exercise of Congress's taxing power, is this tax constitutional? A Yes, because Congress can tax passenger tickets due to its power to regulate interstate commerce. B Yes, because Congress may exercise its power to tax for any public purpose. C No, because the tax impinges on the fundamental right to travel. D No, because direct taxes are subject to the apportionment requirement of Article I, Sec. 2 of the U. S. Constitution.

Answer choice B is correct. As long as a tax has a reasonable relationship to revenue production, Congress may exercise its power to tax for any public purpose. Answer choice A is incorrect because the Congressional power to tax is not limited to activities over which Congress otherwise has the power to regulate, such as interstate commerce. Answer choice C is incorrect because, while there is a fundamental right to interstate travel, not all restrictions on this right are unduly burdensome. Answer choice D is incorrect because, although Article I, Sec. 2 does require that all direct taxes be apportioned among the states based on their population, the tax on tickets for passenger train travel is not a direct tax, but rather an indirect tax.As such, it is not subject to the apportionment requirement.

Congress created an investigatory committee to look into the judicial administrative processes of federal judges. In particular, the committee was tasked with investigating the issuance of court orders, the management of trial schedules, and the drafting of basic court documents by judicial assistants. The goal of the committee was to ensure that the administrative processes were carried out efficiently, accurately, and pursuant to federal legislation regarding the judicial administrative process. After much investigation, the committee concluded that a federal district court judge was negligently mismanaging his trial schedule, causing undue delays in his case log. Based upon its findings, the committee made a recommendation to Congress to remove the federal judge.Can Congress remove the federal judge based on the committee's recommendation? A No, because Congress cannot investigate a federal judge's administrative processes. B No, because the federal judge may only be removed by impeachment. C Yes, because Congress can remove federal judges with or without cause. D Yes, because the federal judge mismanaged his trial schedule.

Answer choice B is correct. Congress is limited in its restraints on the independence of the judiciary, and in particular under Article III, Section 1, as to its ability to remove or impeach a federal judge. Federal judges may "hold their offices during good behavior" and may be removed only by impeachment. Here, impeachment proceedings were not brought against the federal judge. Rather, the removal was sought based on the investigatory committee's recommendation. Moreover, although the federal judge in this case mismanaged his trial schedule, this does not violate the good behavior clause of the Constitution. A violation of the good behavior clause would more likely arise in a scenario where a judge has accepted a bribe or committed perjury. Answer choice A is incorrect because Congress can investigate judicial administrative processes. Answer choice C is incorrect because Congress cannot remove federal judges without proper impeachment proceedings based on a lack of "good behavior." Answer choice D is incorrect because, as stated above, mismanagement of a trial schedule likely does not violate the good behavior clause.

Many citizens are concerned that large contributions by special interest groups and corporations to local politicians' campaigns and recent state constitutional referendums have undermined the state's democratic process. To reduce the danger of corruption and the imbalance in the presentation of arguments posed by excessive contributions, the State legislature passed a law limiting corporate and institutional donors' contributions to candidates and ballot measures to $500.Is the state law constitutional? A No, because corporations and other groups enjoy First Amendment free speech rights. B No, because a law may not limit the contributions to ballot measures. C Yes, because the law is closely drawn to correspond with a sufficiently important state interest. D Yes, because the law is rationally related to the state's legitimate interest.

Answer choice B is correct. Statutes limiting campaign contributions are subject to intermediate scrutiny: they must be "closely drawn" to correspond with a sufficiently important interest. Laws may limit contributions to individual candidates, but not to ballot measures. This law is limiting contributions to ballot measures and is therefore unconstitutional. Answer choice A is incorrect because corporations and other groups do enjoy free speech protections, but states can limit contributions to individual candidates. Statutes limiting campaign contributions are subject to intermediate scrutiny. Reasonable limits on campaign contributions are constitutional because excessive contributions to candidates create a greater "danger of corruption and the appearance of corruption" than do legislatively imposed spending limitations. Answer choice C is incorrect because although it lists the correct standard of review (intermediate scrutiny), the state cannot limit contributions to ballot measures. Answer choice D is incorrect because it gives the wrong test (rational basis), and a state cannot limit contributions to ballot measures.

A privately owned architectural firm incorporated in State A contracted with the federal government to design and construct a federal research facility on federal land in State B. Because the firm wanted to source its construction materials close to the construction site, it purchased most of the needed supplies in State B from State B manufacturers. During the lengthy project, State B passed a law imposing a higher sales tax on industrial construction materials sold in State B. The architectural firm has challenged the constitutionality of the tax as applied to the supplies purchased for construction of the federal facility.Is the tax constitutional as applied to the supplies purchased by the firm for construction of the federal facility? A No, because the federal government and its instrumentalities are immune from state taxation. B No, because states cannot pass legislation retroactively impairing contractual rights and obligations. C Yes, because the private firm is not immune from nondiscriminatory state taxation. D Yes, because the state has the exclusive power to tax intrastate commerce.

Answer choice C is correct. The federal government and its instrumentalities are immune from taxation by the states. However, states may impose generally applicable indirect taxes so long as they do not unreasonably burden the federal government (e.g., state income taxes on federal employees). Here, this generally applicable sales tax on industrial construction materials that applies to a private architecture firm building a federal building is not so unreasonably burdensome as to be an impermissible taxation of the federal government. Answer choice A is incorrect because parties who contract to perform a service for the federal government are not immune from state taxation unless it is somehow so burdensome that it constitutes taxation of the government. Answer choice B is incorrect. State legislation that substantially impairs a contract between private parties is invalid, unless the government can demonstrate that the interference was reasonable and necessary to serve an important governmental interest. This legislation does not substantially impair a contractual right or duty; rather, it makes performance of a contracted duty more difficult, but not prohibitively so. Answer choice D is incorrect because Congress can tax and spend for the general welfare, and this can include taxation of intrastate commerce that has an aggregate effect on interstate commerce.

Where discrimination is the issue, you also need to remember that, in order for a statute to be held invalid, it must be found to be discriminatory in one of the following ways: 3

It must be (1) discriminatory on its face, or be (2) facially neutral but unequally administered, or (3) have an impermissible motive—the intent to discriminate. The statute won't be found invalid if the discrimination results merely from its impact

what is the scrutiny used for age?

Rational Basis § Must be: 1. Rationally related 2. To a legitimate government interest

what is the scrutiny used for Same-sex marriage?

o level of scrutiny unclear. § you should assume that any law that either forbids or materially disadvantages same-sex marriages (compared with opposite-sex marriages) violates equal protection; you will presumably not be asked to say what level of scrutiny is to be used in reviewing such laws.

what does the Fifteenth Amendment prohibit?

prohibits both the state and federal governments from denying any citizen the right to vote on the basis of race, color, or previous condition of servitude. The courts have interpreted the right to vote to include the right to have that vote meaningfully counted.

State actions that discriminate against a suspect class or substantially impact a fundamental right are subject to...

strict scrutiny. Under this most stringent standard of review, such actions are unconstitutional unless the state can prove that they are necessary to advance a compelling state interest.

"Interstate" Privileges & Immunities Clause, Article IV, § 2.

§ It prevents states from discriminating against out-of-state citizens and residents in matters concerning "essential activities" (e.g., pursuing one's livelihood, owning property) and "basic rights" (e.g., medical care, court access), unless the discrimination is closely related to achieving a substantial state purpose (e.g., protecting natural resources by the state) and there are no less restrictive means available to achieve the purpose. It's important to remember that this provision doesn't protect corporations or aliens—just out-of-state, human, U.S. citizens.

Establishment Clause test:

§ the government action must have a secular (i.e., non-religious) purpose; § the primary or principal effect of the action must not be the advancement of religion; and § the government action must not foster excessive governmental entanglement with religion.

what is the scrutiny used for preference of religion?

strict scrutiny o the "compelling interest" test: the classification scheme must be necessary (The least restrictive means) to achieve a compelling governmental interest.

Is Unintentional burdening of religious practices allowable?

. The right to free exercise of religion does not relieve a person of the obligation to comply with a valid and neutral law of general applicability, even though the law forbids (or requires) conduct that the person's religion requires (or forbids).

The First Amendment shields the media from liability for publishing truthful information that was unlawfully obtained by a third party if 2

(1) the information involves a matter of public concern and (2) the publisher neither obtained it unlawfully nor knows who did.

Content-based regulation of commercial speech must survive which scrutiny?

intermediate scrutiny, i.e., must be "significantly related" to achieving an "important" governmental objective. A content-based regulation of speech in a public forum is subject to strict scrutiny.

The success of a First Amendment challenge to a conditional spending provision depends on whether the condition is placed on:

funded activities - in which case the government can insist that the funds be used for authorized purposes (eg, requiring that funds be used only for research and testing) or funding recipients - in which case the government cannot restrict the recipient's right to engage in constitutionally protected conduct that falls outside the funding's purpose or scope.

does the 11th amendment prevent equity suits in federal court where a state official has violated the claimant's federal constitutional rights

NO

does the 11th amendment prevent suits against a state official for money damages, as long as the damages are to be paid out of the official's own pocket.

NO

does the 11th amendment prevent suits by anyone against state subdivisions (e.g., cities or counties)?

NO

May Congress abrogate the Eleventh?

NO except where it passes a statute giving private citizens the right to sue a state under the post-Civil War Amendments. it's limited to measures that are fitted to the remedying of actual constitutional violations by the states.

Are Defamation, obscenity, and "fighting words" considered to be "speech"?

No and thus are not protected by the First Amendment. as long as a statute contains narrow and definite standards, such speech can be prohibited without meeting the "compelling interest" test.

what is the scrutiny used for religion if P tries to excuse himself from compliance and the statute is neutral?

Rational Basis § Must be: 1. Rationally related 2. To a legitimate government interest

State law that discriminate against resident aliens? Federal law that discriminate against resident aliens? scrutiny and rights?

State law: strict scrutiny; Equal protection because it is a classification (14th) Federal law: rational basis - 5th

If there is classification, is the problem due process or equal protection?

equal protection o Where there's no classification it is due process problems.

any statute regulating freedom of speech (or association) must contain.... 2

narrow and definite standards in order to be upheld. This involves the twin doctrines of overbreadth and vagueness. If a statute prohibits not only unprotected speech, but some protected speech as well, it's unconstitutionally overbroad. If the conduct prohibited by a statute is so unclearly defined that a reasonable person would have to guess at its meaning, it's unconstitutionally vague. Note that statutes that are unconstitutionally overbroad are usually unconstitutionally vague, too.

The president of the United States received reliable information from federal law enforcement authorities that (1) a known terrorist group was planning a terrorist attack on America which would occur within the next two weeks, (2) the terrorists, all of whom were fluent in a particular dialect, were already in America, and (3) the terrorist group's leaders would provide to these terrorists certain details regarding the attack through coded messages contained in a U.S. newspaper published in the particular dialect. There were four such newspapers—in New York, Washington, Los Angeles, and Detroit. The president immediately ordered all four newspapers to shut down for two weeks and notified the newspapers that they were to be fully compensated for any losses they incurred because of the closure order. The newspapers immediately challenged the order as unconstitutional. Which of the following is the president's best argument that the order should be upheld? A It is not a prior restraint on speech or the press. B The federal government can always suppress subversive speech as long as it pays just compensation to the person whose expression has been suppressed. C The words that would be published constitute a clear and present danger to national security. D Because national security is at issue, the burden is on the newspapers to establish the right to publish the information.

Answer choice C is correct. Although governmental regulation of speech based on its content is generally prohibited, among the limited exceptions permitting such regulation is speech that represents a clear and present danger of imminent lawless action. Moreover, while a prior restraint on speech or the press is typically presumed to be unconstitutional, and has been rejected even where national security was at issue, it is possible that in the face of an immediate threat of grave and irreparable harm, such as a terrorist attack that would be triggered by messages in the newspapers, a prior restraint on their publication would be upheld. Answer choice A is factually incorrect: the president's order would prevent the four newspapers from publishing for two weeks, and thus constitutes a prior restraint on speech and the press. Answer choice B is incorrect because suppression of subversive speech generally is not permitted unless it presents a clear and present danger, but, if suppression is allowed, there is no requirement, unlike the taking of property under the Takings Clause of the Fifth Amendment, that the government compensate the person whose speech has been suppressed. Answer choice D is incorrect because where a prior restraint, even one based on national security, is imposed on Freedom of Speech, the burden is on the government to justify the restraint.

does the 11th amendment prevent suits by the federal government against states?

no

what is the scrutiny used for right to marry?

strict scrutiny o the "compelling interest" test: the classification scheme must be necessary (The least restrictive means) to achieve a compelling governmental interest.

what is the scrutiny used for speech restriction on the basis of its content?

strict scrutiny o the "compelling interest" test: the classification scheme must be necessary (The least restrictive means) to achieve a compelling governmental interest.

which scrutiny if the court decides to close criminal procedures to the public and the media?

strict scutiny

what is the test for Rational Basis?

§ Must be: 1. Rationally related 2. To a legitimate government interest

what is the test for Intermediate Scrutiny?

§ The government must show it is: 1. Substantially related 2. To an important government interest

what is the president immunity?

Absolute immunity from civil suits seeking damages for official executive actions No immunity for actions that occurred before taking office No immunity for actions unrelated to executive functions No immunity from criminal subpoena

A state seeking to terminate parental rights must prove its case by at least _____________________________- to comply with procedural due process.

clear and convincing evidence

what is the test under substantive due process when there state action?

At a minimum, due process requires that any such government action be rationally related to a legitimate government interest (ie, rational basis review). A government action that is grossly disproportionate to a person's offense or circumstances cannot satisfy this standard of review.

what is the scrutiny used for purely economic and social legislation?

Rational Basis § Must be: 1. Rationally related 2. To a legitimate government interest

If there is no discrimination in a classification, then we use substantive due process

Substantive due process prohibits the government from depriving persons of life, liberty, or property (eg, freedom of movement) without adequate justification.

what is the scrutiny for political speech (which includes campaign donation)?

intermediate scrutiny

what type of classification does the Rational Basis apply?

o Applies to all other rights and classifications

what is a non justiciable case?

o a federal court will dismiss a case that is non-justiciable because they call for § 1) advisory opinions, § 2) are not yet ripe, § 3) are moot, § 4) or involve "political questions." § 5) Do not have standing § 6) Do not involve a case or controversy

what type of classification does the Intermediate Scrutiny apply?

o gender and legitimacy (non-marital children born outside of marriage)

state law discriminating against non US citizen, which scrutiny?

State laws that discriminate against resident aliens are generally subject to strict scrutiny. But the rational basis test will be used if the political-function exception applies—ie, the law (1) excludes resident aliens from participating in an important government function and (2) is sufficiently tailored to the state's interest in limiting participation.

test for validity of government action when there is establishment of religion clause?

The government may interact with religious institutions so long as it remains neutral towards religion. The government complies with the First Amendment establishment clause if its conduct satisfies all three prongs of the Lemon test: (1) the conduct has a secular purpose, (2) its primary effect does not advance or inhibit religion, and (3) it does not create excessive government entanglement with religion.

analysis of commerce clause If there is Federal law?

o supremacy clause - does federal preempts state? § State direct contradicts federal -> preemption § No direct contradiction: did congress intend Federal law to occupy the entire field? Look at: (1) whether the subject matter is traditionally classified as local or federal; (2) how pervasive the federal regulation is; (3) how similar the state and federal laws are (the more they coincide, the more likely it is that federal law was intended to supersede state law); and (4) whether there's a need for uniform federal regulation.

what is the test for strict scrutiny?

o the "compelling interest" test: the classification scheme must be necessary (The least restrictive means) to achieve a compelling governmental interest.

what is the scrutiny used for Programs that favor racial or ethnic minorities

strict scrutiny o the "compelling interest" test: the classification scheme must be necessary (The least restrictive means) to achieve a compelling governmental interest.

what is the scrutiny used for liberty to control children's education?

strict scrutiny o the "compelling interest" test: the classification scheme must be necessary (The least restrictive means) to achieve a compelling governmental interest.

what's Enclave Clause?

· Congress has the general police and regulatory powers over the District of Columbia that a state enjoys over persons and things within its boundaries.

what's the ministerial exception?

· The Free Exercise Clause protects a religious organization's right to shape its own faith and mission through its choice of ministers. The Establishment Clause prevents the government from imposing its choice of a minister on the organization. Together, these clauses create a ministerial exception that operates as an affirmative defense to an employment discrimination claim brought by a minister against the church.

HOW TO ANALYSE THE VALIDITY OF A STATUTE

1) A.Determine first if the statute is a federal statute or a state statute. 2) B.Analyze the validity of federal statutes or actions. a. valid federal statute must be rationally related to an enumerated power, or it must be necessary and proper to effectuate an enumerated power. b. 3) C.Determine the validity of state statutes. a. 1.The law must be enacted within the state's powers (e.g., police powers); i. police power: involves the public health, safety, welfare, or morals. b. 2.It must not violate any person's constitutional rights; and i. The two most frequent ways in which a state statute can violate the Constitution involve due process and equal protection. c. 3.It must not unduly burden interstate commerce. i. Determining if an undue burden exists requires the application of a balancing test: Is the burden on interstate commerce outweighed by legitimate interests of the state in protecting its citizens, taking into account less burdensome alternatives? Also, the regulation must be non-discriminatory, unless the state has no reasonable, non-discriminatory alternatives in its effort to protect health and safety.

level of scrutiny for speech restriction when it is 1) content-based and 2) content-neutral?

1) content-based - in which case, strict scrutiny applies and the law is unconstitutional unless the government can show that it is necessary and narrowly tailored (ie, least restrictive means) to achieve a compelling government interest or 2) content-neutral - in which case, intermediate scrutiny applies and the law is permissible so long as the government can show that it is narrowly tailored to achieve an important government interest and leaves open alternative channels of communication.

For voting rights, What is the burden: 1) When an electoral regulation imposes an ordinary burden? 2) if the burden is severe?

1) the challenging party must prove that the regulation is not rationally related to a legitimate state interest (ie, rational basis test). 2) the state must show that the regulation is the least restrictive means of achieving a compelling state interest (ie, strict scrutiny).

§ state or local regulation that, on its face or in practice, discriminates against out-of-state commerce is unconstitutional unless it meets the following requirements to be upheld:

1. (1)The regulation must pursue a legitimate end; 2. (2)The regulation must be rationally related to that legitimate end; and 3. (3)The regulatory burden imposed by the state on interstate commerce, and any discrimination against interstate commerce, must be outweighed by the state's interest in enforcing the regulation.

What's the four-part test a restriction on commercial speech must survive?

1. (i) it must concern lawful activity and be neither false nor misleading; 2. (ii) the asserted government interest must be substantial; 3. (iii) the regulation must directly advance the asserted interest; and 4. (iv) the regulation must be narrowly tailored to serve that interest.

what is the scrutiny used for Legitimacy

1. Intermediate scrutiny. o Test: a law must be substantially related to an important state interest in order to be valid. § The government must show it is: 1. Substantially related 2. To an important government interest

Congress enacted a law establishing a federal life insurance exchange under which United States citizens and residents, regardless of age, can acquire affordable life insurance. Although United States citizens are not subject to a residency requirement in order to participate in the exchange, resident aliens are required to have lived in the United States for at least five years to be eligible. A resident alien who has lived in the United States for four years has filed suit in federal court, claiming an equal protection violation. The government has asserted only that the law is rationally related to a government interest, and the court has indicated that it agrees.Is the resident alien likely to prevail? A No, because Congress has plenary power over aliens under Article I of the Constitution. B No, because the law violates the Privileges or Immunities Clause of the Fourteenth Amendment. C Yes, because national origin is a suspect classification that triggers strict scrutiny. D Yes, because the government failed to show that the law is substantially related to an important government interest.

Answer choice A is correct. A classification based on alienage is subject to a different standard depending on whether the action is taken by the state or by the federal government. Courts will generally apply the strict scrutiny test and strike down state-based laws that discriminate against resident aliens for lack of U.S. citizenship. In contrast, Congress has plenary power over aliens under Article I, and the power to expel or exclude aliens is a fundamental sovereign attribute exercised by the Government's political departments largely immune from judicial control. Therefore, a federal alienage classification is likely valid unless it is deemed arbitrary and unreasonable. Here, the government proved that the law was rationally related to a legitimate governmental interest and thus the resident alien's claim will fail. Answer choice B is incorrect because the Privileges or Immunities Clause of the Fourteenth Amendment does not apply to aliens but only to United States citizens; further, this clause applies only to the states, not the federal government. Answer choice C is incorrect because national origin isn't at issue in this question; the issue is one of alienage. Answer choice D is incorrect because intermediate scrutiny applies to quasi-suspect classifications based on gender and nonmarital child status, but not to a federal alienage classification.

A defendant in a criminal trial wore a shirt depicting a political symbol. The judge ordered the defendant to remove the shirt and wear it inside out, explaining that the clothing constituted a political statement that could improperly influence the jury and thereby undermine the fairness of trial. When the defendant, contending that he had a right to make a political statement, refused, the judge held the defendant in contempt.Can the defendant successfully challenge the contempt order on the grounds that the court's order to remove his shirt violated his First Amendment Free Speech Clause? A No, because the courtroom is a non-public forum. B No, because the wearing of clothing is not protected by the First Amendment Free Speech Clause C Yes, because the order to remove the shirt was based on the content of the speech on the shirt. D Yes, because wearing the shirt constituted political speech.

Answer choice A is correct. A nonpublic forum is essentially all public property that is not a traditional or designated public forum, and encompasses government offices, schools, jails, military bases, and polling places. The government may regulate speech-related activities in nonpublic forums as long as the regulation is (i) viewpoint-neutral and (ii) reasonably related to a legitimate governmental interest. Here, the courtroom is classified as a nonpublic forum for purposes of the First Amendment Free Speech Clause. While the judge specifically banned the defendant from wearing the shirt because of its political content, the judge did not ban the shirt on the basis of its specific political point of view. Therefore, the decision was content-based, but was also viewpoint-neutral. In addition, the judge banned the shirt because its political statement could undermine the fairness of the trial, which is reasonably related to a legitimate governmental interest. Answer choice B is incorrect because wearing a shirt with a political symbol is expressive conduct, which enjoys protection as speech. Answer choice C is incorrect because, although the judge's order to the defendant to remove his shirt was directed at the shirt's political content, speech in a nonpublic forum may be subject to content-based governmental regulation provided that the regulation is viewpoint-neutral and reasonably related to a legitimate governmental purpose. Answer choice D is incorrect because, although political speech lies at the heart of the First Amendment free speech protections, it may be subject to governmental regulation in a nonpublic forum, such as a courtroom. LEARN WHYNEXT QUESTION

A state required its political parties to allow every registered voter in the state to vote in party primaries. A newly-formed state party wanted to hold a primary to pick its presidential electors, but the national party with which the state party was affiliated required that electors be chosen only by party members. A neighboring state prohibited independents from voting in party primaries. A long-established third party hoped to finally secure the governor's mansion, which it felt hinged on the ability to attract a large percentage of the independent voters. Both the newly formed state party and the established third party challenged their respective states' primary regulations in federal court as unconstitutional.What will be the outcome of these lawsuits? A Both the newly-formed state party and the established third party will prevail. B The newly-formed party will prevail, but the established third party will lose. C The established third party will prevail, but the newly-formed party will lose. D Both the newly-formed party and the established third party will lose.

Answer choice A is correct. A state cannot require a local party to participate in an open primary (i.e., a primary in which any voter in the state may vote in a political party's primary) to choose presidential electors where the national party has required that electors must be chosen only by party members. In addition, a state may not prohibit a political party from allowing independents to vote in its primary. Answer choice B is incorrect because states cannot prohibit political parties from allowing independents to vote in primaries. Answer choice C is incorrect because open primaries cannot be required by the state. Answer choice D is incorrect because both statutes are unconstitutional. (Note: The length of time that the political party has been in existence is irrelevant for this purpose.)

Pursuant to a valid delegation of power by Congress, the United States Department of Transportation (DOT) promulgated a safety standard that permitted, but did not require, driverless car programmers to include one of several specific types of crash-prevention subprograms in order to encourage the further development of these subprograms. A plaintiff who was injured when struck by a driverless car initiated a negligence suit under state common law against the software company that wrote the program controlling the driverless car at the time of the accident. The claim rested on the allegation that the specific crash-prevention subprogram included by the defendant software company in its driverless car program was defective, even though that subprogram was one of the types permitted by the DOT safety standard. The state court ruled that, based on the Supremacy Clause of Article VI of the United States Constitution, the plaintiff's common-law action was preempted by the DOT safety standard.Of the following, which provides the best support for the court's ruling? A A successful suit would thwart the stated purpose of the DOT safety standard to encourage the further development of crash-prevention subprograms. B Federal statutes and regulations set a ceiling above which state law cannot go. C The federal regulation that permitted use of this crash-prevention subprogram must be broadly construed as expressly prohibiting state laws that punish the use of the subprogram. D There is presumption that a federal law or regulation preempts state law unless Congress expressly states otherwise.

Answer choice A is correct. A state law may indirectly conflict with federal law by creating an obstacle to or frustrating the accomplishment of that law's purpose. Here, by allowing a state common law action against the defendant software company, the purpose of the federal regulation to encourage further development of crash-prevention subprograms would be thwarted. Answer choice B is incorrect because federal statutes and regulations set a floor below which state law cannot go, but a state is free to set more stringent standards on conduct than those imposed by the federal government as long as it does not infringe on a federally protected right. Answer choice C is incorrect because even a federal law or regulation that expressly preempts state law must be narrowly construed, not broadly construed. Here, the federal regulation does not expressly prohibit state laws that penalize the use of a defective crash-prevention subprogram. Answer choice D is incorrect because it misstates the presumption. There is a presumption that a federal law does not preempt state law unless Congress expressly states otherwise. LEARN WHYNEXT QUESTION

A medical supply company made transfers of property to a state hospital after the company was insolvent. Subsequently, the company filed for bankruptcy in federal bankruptcy court. The bankruptcy trustee, acting pursuant to the federal Bankruptcy Act, sought to recover those transfers. The state hospital, refusing to comply with the trustee's request, asserted that, as a state entity, it was immune from suit by a private individual, despite a provision in the Bankruptcy Act that abrogated state immunity with respect to such transfers. Can the trustee force the state to turn over the transferred property? A Yes, because Congress was acting pursuant to the Bankruptcy Clause of Article I, Section 8, Clause 4, in subjecting the state to the Bankruptcy Act. B Yes, because the action was brought in federal court. C No, because the Eleventh Amendment protects a state entity from suit by a private individual for damages in federal court. D No, because Congress lacks the power to abrogate state immunity.

Answer choice A is correct. Although Congress, acting pursuant its Article I powers, generally cannot abrogate state immunity, the Bankruptcy Clause gives Congress the power to subject states to its provisions. While the Eleventh Amendment generally immunizes the state from suits in federal court for money damages or equitable relief when the state is a defendant in an action brought by a citizen of another state, it does not bar the actions of a bankruptcy court that impacts state finances. Answer choice B is incorrect because the bringing of a lawsuit in federal court does not automatically subject a state to liability. In fact, a state is generally protected from a lawsuit brought in a federal court by a private individual for damages under either the Eleventh Amendment or the doctrine of sovereign immunity. Answer choice C is incorrect because, despite the Eleventh Amendment's protection, Congress can subject a state to the provisions of the Bankruptcy Act. Answer choice D is incorrect because Congress does have the power to abrogate state immunity if it does so clearly and acts pursuant to two different powers. The first is bankruptcy. The second is the enforcement of rights created by the remedial provisions of the Thirteenth, Fourteenth, and Fifteenth Amendments (i.e., the Civil War Amendments).

Concerned with the federal budget deficit, Congress enacted a national property tax on all real property interests within the United States. A fixed rate is applied uniformly throughout the United States to the fair market value of these interests.Is this tax constitutional? A No, because of Article I, Section 9 of the Constitution, concerning direct taxes. B No, because taxes on real property interests are historically a matter left to state and local governments. C Yes, because it is uniformly applied throughout the United States. D Yes, because of the Sixteenth Amendment to the Constitution.

Answer choice A is correct. Article I, Section 9 provides that "no . . . direct tax shall be laid, unless in proportion to the Census[.]" Although there is dispute as to the types of taxes that are encompassed within the definition of a direct tax, a tax on real property interests is undisputedly a direct tax. To satisfy the apportionment rule, a state with twice the population of another state would have to pay twice the tax, even if the more populous state's share of the national tax base were smaller. Here, because the real property tax is not in proportion to the population of each state, it would run afoul of Article I, Section 9. Answer choice B is incorrect. Although it is true that local governments historically have imposed a tax on real property interests, Article I, Section 8 gives Congress the power to raise revenue through the imposition of taxes, which is the express purpose of this tax. Answer choice C is incorrect. Article I, Section 2 requires that "direct taxes shall be apportioned among the several states." This uniformity requirement has been interpreted as requiring geographical uniformity, meaning that the product or activity at issue must be identically taxed in every state in which it is found. However, this does not negate the requirement that states must be taxed by direct taxes in proportion to their populations. This direct tax makes no attempt to ensure that each state will be taxed proportionally to its population. A tax on all real property interests within the United States would not satisfy this requirement, even if it is uniformly applied throughout the United States. Answer choice D is incorrect. Although the Sixteenth Amendment allows a tax without apportionment among the states, the Sixteenth Amendment is limited to a tax on income and does not apply here.

A city owned and operated a minor-league baseball park. The city sold advertising space on billboards above the park and along the walls of the baseball field. While most of the advertising space was purchased by beer and snack food companies that also sold their goods at the baseball games, a city ordinance allowed for the sale of space to political, charitable, and religious causes. All final decisions on the advertising were solely at the discretion of a particular city official. A modern-day temperance organization wanted to buy space on some of the billboards to warn of the dangers associated with drinking alcohol. The city official wanted the baseball games to remain light-hearted and fun events for the citizens, and also did not want to risk the beer companies pulling their advertisements from the ballpark. Accordingly, he denied the temperance organization's request for advertising space. The temperance organization sued the city and the official, claiming that the denial of the organization's request was unconstitutional.Is the temperance organization likely to succeed in its suit? A Yes, because the city official may not deny an organization's right to broadcast its message in a public forum on the basis of its content unless the denial is necessary to serve a compelling government interest. B Yes, because the city official may not refuse to allow an organization the use of a public facility to broadcast a message dealing with an issue of public concern. C No, because the city official's denial of advertising space was a reasonable time, place, and manner restriction. D No, because a valid city ordinance gave the official discretion to grant or deny advertising space.

Answer choice A is correct. Because the city opened the baseball park to advertisers of all kinds, the city-owned baseball park is a public forum. A content-based regulation of speech in a public forum is subject to strict scrutiny. Thus, the regulation must be necessary to achieve a compelling government interest, and narrowly tailored to meet that interest. Preserving the light-hearted nature of a baseball game and preventing the possible loss of other advertising revenue is likely not a compelling interest. Answer choice B is incorrect because it is too broad. Not all public facilities qualify as public forums. Furthermore, a government official may regulate speech if the regulation is narrowly tailored to achieve a compelling government interest. Answer choice C is incorrect because time, place, and manner restrictions in a public forum are subject to strict scrutiny, and the actions in this case do not meet this standard. Answer choice D is incorrect because a city ordinance, whether validly enacted or not, may not give a city official the power to unconstitutionally restrict free speech. LEARN WHYNEXT QUESTION

A leader of an extremist cult was charged with murdering a fellow cult member as part of a human sacrifice at the building he owned and used as the cult's compound. The leader was detained in jail before raising the money necessary to pay his bond. While in jail, the leader received notice that although the investigation of the compound had been completed, the government planned to seize the property on a given date. When he was released on bond after the date indicated in the notice, the leader returned to the compound. When he arrived, he found that the compound had been abandoned and boarded up. A notice on the boarded-up door indicated that even though no evidence remained on the property, the compound had been seized by the government pending the resolution of his trial.Have the leader's due process rights been violated? A Yes, because the government seized his property without a hearing. B Yes, because the government seized his property while he was in jail. C No, because the leader was provided with notice of the pending seizure. D No, because the leader was not entitled to a hearing prior to seizure.

Answer choice A is correct. Forfeiture is an involuntary relinquishment of property that the government alleges is connected to criminal activity. Generally, the government is required to provide the owner with notice and a hearing prior to seizure of real property. Here, because the cult leader was denied a hearing before the seizure of his compound, his due process rights have likely been violated. Answer choice B is incorrect because the fact that the leader was in jail when the property was seized is not determinative of whether his due process rights were violated. Answer choice C is incorrect because notice of the seizure without an opportunity for some sort of hearing is insufficient due process for a seizure of real property. Answer choice D is incorrect because the leader was entitled to a hearing prior to the seizure of the compound as such process is required prior to the seizure of real property. LEARN WHYNEXT QUESTION

The jury commissioner at a state court was charged with stealing the identities of dozens of potential jurors. She readily confessed to her crime to her employer immediately after the allegations of her behavior surfaced. The county immediately terminated her employment. Both her employment contract and state law clearly stated that jury commissioners can be terminated only if there is just cause to do so. A hearing was scheduled for a week following the commissioner's termination. The commissioner filed suit to have herself reinstated as commissioner. She alleged that, because she had not been convicted, she was entitled to a pre-termination notice and an opportunity to respond.Has the commissioner likely received the due process to which she is entitled? A Yes, because she will receive a prompt post-termination hearing. B Yes, because no hearing is required when there is cause to terminate the employee. C No, because she has not been convicted of a crime. D No, because she did not receive a pre-termination hearing.

Answer choice A is correct. Generally, a public employee who may be discharged only for cause has a property interest in her job, and therefore is entitled to notice of termination and a pre-termination opportunity to respond. A formal pre-termination hearing, however, is not required. If there is a significant reason for immediately removing a "for-cause" employee from the job, a prompt post-termination hearing with reinstatement and back pay if the employee prevails constitutes sufficient due process. Here, the commissioner's theft of information directly related to her position as jury commissioner and the ability to continue such criminal behavior is a justification for her immediate removal as commissioner. Accordingly, only a post-termination hearing is required, and as that hearing is set for the week following the commissioner's termination, the commissioner has likely received proper due process. Answer choice B is incorrect because, even though there is justification for the immediate removal of the commissioner, she is still entitled to a post-suspension hearing. Answer choice C is incorrect because, although the conviction of a crime would constitute just cause for removal, her contract only requires just cause, which is satisfied by her confession to committing the identity thefts. Answer choice D is incorrect because no pre-termination hearing is required in light of the justification for immediately removing the commissioner from her position.

As part of a woman's religion, she was required to burn a small amount of holy oil in an urn before consuming a meal. Although she was able to burn the holy oil before breakfast and dinner at home, she needed to conduct this ritual at work before her lunch break. The woman's supervisor asked her to burn the holy oil outside, but because she had several other religious accoutrements that she needed to use in conjunction with the ritual, she insisted on burning the oil in her cubicle. In addition, the woman stated that she had a flame snuffer in order to ensure that the fire did not pose a danger. A state fire statute prohibits any open fires in public buildings because they present a fire hazard. The woman has filed an action in state court challenging the constitutionality of this statute, claiming that it violates her right to the free exercise of religion.Assuming no relevant federal or state statute is applicable, should the court rule that the state statute violates the Free Exercise Clause of the First Amendment? A No, because preventing fires inside of public buildings is rationally related to a legitimate government interest. B No, because the state statute does not result in excessive entanglement with religion. C Yes, because preventing a religious ritual of this type does not advance a compelling state interest. D Yes, because the statute is not substantially related to an important governmental interest.

Answer choice A is correct. Neutral state laws of general applicability that have the incidental effect of interfering with one's ability to engage in religious practices are subject only to the rational basis test. A law passes the rational basis test if it is rationally related to a legitimate state interest. Here, the state statute prohibiting fires in public buildings is a neutral, generally applicable statute that is rationally related to the legitimate state interest of preventing fires because they pose a great safety risk. Therefore, the woman is unlikely to succeed in her action. Answer choice B is incorrect because that standard is one of the three-prongs of the Lemon test that has been applied to violations of the Establishment Clause, not the Free Exercise Clause. Answer choices C and D are incorrect because they apply tests (i.e., strict and intermediate scrutiny) that do not apply to laws that have only an incidental impact on the free exercise of religion. LEARN WHYNEXT QUESTION

A professor was granted tenure at a specific state university. As a tenured faculty member, he was entitled to continued employment at that university until retirement or resignation unless he was terminated for good cause. For the next 10 years, he taught at the university. Then, facing a budget deficit, the state legislature abolished that specific university. Consequently, after receiving notice, the professor was terminated without any hearing.Have the professor's procedural due process rights been violated? A No, because his termination was not an adjudicative act. B No, because the professor received notice of his termination. C Yes, because he had tenure. D Yes, because he was terminated without a hearing.

Answer choice A is correct. Procedural due process only applies in quasi-judicial or adjudicatory settings, and not with respect to the adoption of general legislation. Here, the professor was not terminated as an individual, but instead as a consequence of the abolition by the legislature of the specific university at which he was granted tenure. Therefore, his termination did not trigger his procedural due process rights. Answer choice B is incorrect. Generally, procedural due process requires both notice and a hearing when a person is deprived of life, liberty, or property. Here, although the professor was deprived of a property interest (i.e., his tenured position at a specific university), the deprivation was not the result of an adjudicatory process, but instead a legislative one. As such, as a matter of procedural due process, he was entitled to neither notice nor a hearing regarding his termination. Answer choice C is incorrect because, although his tenure meant that the professor had a property right in his employment at the university, his termination was not the result of an adjudicatory process, but instead a legislative one. As such, as a matter of procedural due process, he was entitled to neither notice nor a hearing regarding his termination. Answer choice D is incorrect because, although the professor was terminated without a hearing, since his termination was a consequence of a legislative act rather than an adjudicatory act, his termination did not violate his procedural due process rights.

In a state known for its game fish, there are many guide-led fishing expeditions marketed to tourists. The state enacted a statute that required all fishing guides who charge a fee to have a license. The purpose of the statute is to protect the state's game fish from overfishing. The license costs $100 for in-state residents and $300 for out-of-state residents. If an out-of-state resident challenges the constitutionality of this statute, what is the most likely result? A The statute will be struck down under the Privileges and Immunities Clause of Article IV, Section 2. B The statute will be struck down under the Equal Protection Clause. C The statute will be upheld because engaging in fishing is not a fundamental right. D The statute will be upheld because regulation of fishing is traditionally a state, rather than national, function.

Answer choice A is correct. The Article IV Privileges and Immunities Clause prohibits a state from discriminating against nonresidents with respect to the exercise of a fundamental right or engagement in an essential activity, such as earning a living, unless there is substantial justification for the discrimination. Here, the state imposed a fee on nonresidents that was three times greater than the fee on residents. While nonresidents may contribute to the problem that the state was seeking to address (i.e., overfishing), there are other means of addressing this problem, such as limiting the amount of fish that can be caught, that are less restrictive on the rights of nonresidents. Answer choice B is incorrect because, since nonresident fishing guides are not members of a suspect classification, the rational basis standard is applied to determine whether the statute violates the Equal Protection Clause. Under this standard, the statute is likely to be upheld since indirectly limiting the number of guides could reduce the number of individuals who go fishing and thereby the number of fish caught. Answer choice C is incorrect because, although engaging in fishing is not a fundamental right, the right to earn a livelihood is an essential activity for purpose of the Article IV Privileges and Immunities Clause. Answer choice D is incorrect because, even if the state has traditionally regulated fishing, the state cannot do so in a manner that violates individual rights set forth in the Constitution. The conflict here is not between a state statute and a federal statute, but between the state statute and the Constitution.

A coastal state bordering a foreign country engages in more commercial activity with the foreign country than it does with bordering states in this country. The coastal state's legislature, to encourage purchases within this country, enacts legislation increasing the tax on produce purchased from the foreign country. The costs associated with the sale of produce between the countries become substantially higher in light of the tax.Is the taxation proper? A No, because all international taxation falls under the sole authority of Congress. B No, because the tax provides a commercial advantage to local businesses. C Yes, because the state has a legitimate interest in protecting workers in this country. D Yes, because there is a substantial nexus between the taxation of importing fruit and the state's ability to regulate commerce.

Answer choice A is correct. The Import-Export Clause of Article I, Section 10 prohibits the states from imposing any tax on any imported or exported goods, or on any commercial activity connected with imported goods. The Commerce Clause limits states from taxing commerce with other countries because it vests in Congress the exclusive power to regulate foreign commerce. Therefore, the statute would conflict with Congress' exclusive right to regulate foreign commerce. Answer choice B is incorrect because while the statement is true, it does not capture the fundamental basis for the improper nature of the taxation: a state cannot tax imports. Answer choice C is incorrect because although the state does have a legitimate interest in protecting its workers, it cannot accomplish that goal by providing a direct commercial advantage to local businesses and cannot tax the import of goods. Answer choice D is incorrect because whether there is a substantial nexus between the taxed activity and the state is only one consideration when determining whether the taxation is allowed in general circumstances. This incorrect answer choice does not consider the discriminatory nature of the tax, nor does it consider the constitutional prohibition on state taxation of imports. LEARN WHYNEXT QUESTION

A recently enacted federal statute bans the sale of automatic firearms to anyone under the age of 16. A national manufacturer and seller of automatic firearms was charged with violating this statute for selling an automatic firearm to a 15-year-old boy with his parents' consent.Which of the following provides the strongest constitutional argument in the manufacturer's defense against this prosecution? A The statute denies individuals under the age of 16 a fundamental constitutional right. B The statute infringes on the police power reserved to the states by the Tenth Amendment. C The statute is an improper use of the congressional war and defense powers. D The statute unduly burdens interstate commerce.

Answer choice A is correct. The Second Amendment guarantees an individual's right to possess a firearm unconnected with service in a militia and to use that firearm for traditionally lawful purposes. The Second Amendment right to bear arms is not unlimited. Examples of lawful regulations include imposing conditions and qualifications on the commercial sale of arms, as well as prohibitions on (i) concealed weapons, (ii) possession of firearms by felons and the mentally ill, and (iii) carrying guns in schools, government buildings, and other sensitive places are presumed to be legitimate. However, because it is a recognized fundamental right, this argument will force the court to apply strict scrutiny to the regulation in question. Therefore, because applying strict scrutiny will provide the manufacturer with its best chance of finding that this statute is unconstitutional, this is the manufacturer's strongest defense. Answer choice B is incorrect. Banning firearms to those under age 16, in and of itself, is unlikely (even in the aggregate) to substantially affect interstate commerce. Nonetheless, the sale of any product (including guns) is "commerce"; all gun sales nationally certainly have a substantial effect on interstate commerce. The Necessary and Proper Clause allows Congress to ban gun sales to minors as part of its broader regulation of gun sales. Therefore, because this statute regulates commercial activity, it likely falls within the power of Congress to regulate interstate commerce and thus likely does not infringe upon the police powers reserved to the states. Answer choice C is incorrect because, even though this statute is unlikely to be permitted under the congressional war and defense powers, this argument ignores the fact that this statute regulates commercial activity. Therefore, this is not the manufacturer's strongest argument. Answer choice D is incorrect. Under the Dormant Commerce Clause, a state generally cannot enact legislation that unduly burdens interstate commerce. There is no such limitation on congressional authority over interstate commerce. LEARN WHYNEXT QUESTION

During wartime, a number of military aircraft factories expanded their operations and hired many additional employees. As a result of this increased employment, these communities experienced a sudden and unprecedented need for affordable rental housing. In response to this need, Congress passed a law establishing the maximum rent that may be imposed for a two-bedroom apartment within a ten-mile radius of a military aircraft factory.Is this statute likely to be constitutional? A Yes, pursuant to the congressional war and defense powers provided by Article I, Section 8. B Yes, because the statute is rationally related to a legitimate governmental interest. C No, because the statute impermissibly regulates private conduct. D No, because Congress is not authorized to legislate for the general welfare.

Answer choice A is correct. The authority granted to Congress under the war power of Article I, Section 8, is very broad. Congress may take whatever action it deems necessary to provide for the national defense in both wartime and peacetime, including rent control of the civilian economy during wartime, and even during the post-war period. Therefore, this statute is constitutional pursuant to the congressional war and defense powers. Answer choice B is incorrect because rational basis analysis does not apply to legislation passed pursuant to congressional war and defense powers. Answer choice C is incorrect because rent control is not an impermissible limitation on private conduct when enacted under the congressional war and defense powers. Answer choice D is incorrect. Although the General Welfare Clause gives Congress broad power in exercising its spending and taxing powers, it does not give Congress the specific power to legislate for the public welfare in general. However, Congress may rely on other powers to pass legislation that supports the general welfare as long as the legislation also falls under that power.

Compared to traditional incandescent light bulbs, energy-efficient light bulbs typically use approximately 25%-80% less energy. Although most manufacturers of light bulbs have switched to making energy-efficient light bulbs, manufacturers of light bulbs in 10 states still only make traditional incandescent light bulbs. In order to discourage the use of traditional incandescent bulbs, which accelerates the negative consequences of climate change and results in more waste in landfills, Congress imposed a tax on the sale of traditional light bulbs by any manufacturer. The stated purpose of the tax is to generate revenue to help further research to prevent climate change.Is this tax constitutional? A Yes, because there is geographical uniformity in the application of the tax. B Yes, because the tax is necessary and proper to further a compelling governmental interest. C No, because the tax exceeds the point at which financial pressure turns into unconstitutional compulsion D No, because this direct tax will not be apportioned evenly among the states.

Answer choice A is correct. The requirement that indirect federal taxes (i.e., duties, import and excise taxes) must be uniform throughout the United States has been interpreted to mean geographical uniformity only; the product or activity at issue must be identically taxed in every state in which it is found. Here, the traditional light bulbs are being identically taxed, even if the only manufacturers of them are in ten states. Answer choice B is incorrect because the government has no burden to prove that a tax is necessary to any compelling governmental interest. Instead, the General Welfare Clause has been interpreted as permitting Congress to exercise its power to tax for any public purpose. Answer choice C is incorrect because this limitation does not apply to coercion of private actors. Answer choice D is incorrect because sales taxes are indirect taxes, and are not subject to an apportionment requirement.

The mother and father of a young child live together with the child and the child's maternal grandmother in a family home owned by the father. The mother dies, and the father, who is a fit parent, demanded that the child's maternal grandmother leave the family home. The child suffers from a medical condition that, while not life-threatening, demands care that the grandmother, who is a nurse, can provide in a manner that is better for the child than the father alone could provide. The grandmother has filed an action for an injunction compelling the father to permit the grandmother to remain in the family home for the sake of the child for several years until the child outgrows the condition. Her action is based on a state statute that determines issues regarding child care and custody on the "best interest of the child" standard.Does substantive due process provide a ground upon which the court may grant the injunction? A No, because parents have a fundamental right to make decisions regarding the custody and care of their children. B No, because the father owns the family home. C Yes, because related persons, including extended family members, have a fundamental right to live together in a single household. D Yes, because there is a rational basis for permitting the grandmother to remain in the family home to care for the child.

Answer choice A is correct. The right to make decisions regarding the care, custody, and control of one's children is a fundamental parental right. This right has been held to trump the right of a grandparent to have contact with a grandchild in the face of parental opposition. Answer choice B is incorrect. Since the father's property interest is not a fundamental right, the court would need only a rational basis to overcome this property right and grant the injunction, which exists since the grandmother can provide care for the child's medical condition. However, this disregards the issue of the father's fundamental parental right. Answer choice C is incorrect because, while related persons, including extended family members, have a fundamental right to live together in a single household, this fundamental right arises when the government is seeking to force the related persons to live apart. Here, it is the father, not the government, who is seeking to oust the grandmother from the family home. Since the grandmother is seeking an injunction, she is asking the government to compel family members to live together, which does not trigger this fundamental right. Answer choice D is incorrect because, under substantive due process, a rational basis is insufficient grounds for a government to prevent a person from exercising a fundamental right. Here, the father has a fundamental right to make decisions regarding the care, custody, and control of his child. Therefore, a rational basis for permitting the grandmother to remain in the family home to care for the child is insufficient to allow this injunction. LEARN WHYNEXT QUESTION

Congress enacted a statute that made it illegal for "any employee, without the consent of his or her employer, to post on the Internet any information concerning the employer." The purpose of the statute was to prevent employees from revealing their employers' trade secrets.Is the statute constitutional? A No, because it is not narrowly tailored to further a compelling government interest. B No, because it targets a particular medium of communication for special regulation. C Yes, because it leaves open ample alternative channels of communication. D Yes, because it prevents employees from engaging in unethical conduct.

Answer choice A is correct. The statute violates the freedom of speech protected by the First Amendment. The statute targets speech based on its content, because it prohibits employees from posting only "information concerning the employer" on the Internet. Because the statute is a content-based restriction on speech, it is subject to strict judicial scrutiny. Speech restrictions rarely survive strict scrutiny; the government must prove that the restriction is necessary to further a compelling government interest. Even if the government's interest in preventing employees from revealing trade secrets were deemed compelling, Congress could enact legislation utilizing less speech-restrictive means to protect trade secrets. Answer choice B is incorrect because the statute does target one medium of communication--Internet postings--and this focus may cause a court to look more closely at the restriction when evaluating its constitutionality. However, a statute does not violate the First Amendment simply because it targets a particular medium. In this case, the statute violates the freedom of speech protected by the First Amendment because it targets speech based on its content. Answer choice C is incorrect because the statute leaves open channels of communication other than the Internet, but this fact does not save the statute. The availability of ample alternative channels of communication is an element of the First Amendment test for evaluating speech restrictions that are content-neutral, but it is not as important with respect to content-based restrictions. Answer choice D is incorrect. The statute may prevent employees from engaging in unethical conduct, but this fact does not save the statute. Even if the government's interest in preventing employees from revealing trade secrets were deemed compelling, Congress could enact legislation utilizing less speech-restrictive means to protect trade secrets.

A recently enacted federal statute requires the President to make each appointment of a United States ambassador to a foreign country from a list of three individuals. The list is to be compiled by the Senate Foreign Relations Committee and approved by the full Senate in advance of the appointment. The statute also provides that Senate confirmation of the appointment is deemed to occur automatically 30 days after the time the President names an appointee from the list, unless the full Senate determines otherwise within the 30-day period.Is this statute constitutional? A No, because the statute violates the constitutional requirements for appointment of principal officers of the United States. B No, because the statute impermissibly restricts the plenary foreign affairs powers of the President. C Yes, because the statute is consistent with the constitutional requirement that the presidential appointment of ambassadors be with the advice and consent of the Senate. D Yes, because the statute is a necessary and proper measure in furtherance of Congress's power to regulate commerce with foreign states.

Answer choice A is correct. Two provisions of the statute violate the appointments clause of the Constitution as they go beyond simply 'advice and consent'. First, the provision limiting the President to a list of three potential nominees violates the President's power to nominate principal officers. Second, the automatic confirmation provision violates the requirement that the Senate consent to the appointment of a principal officer. Answer choice B is incorrect because the statute does not restrict the President's conduct of foreign affairs. It applies only to the appointment of ambassadors. Answer choice C is incorrect because the statute represents a misapplication of the Senate's advice and consent power. Answer choice D is incorrect. The statutory scheme for the nomination and confirmation of ambassadors is not "necessary" because it is not reasonably related to furthering any congressional statute regulating foreign commerce. Moreover, the statutory scheme is not "proper" because two provisions of the statute described above violate the appointments clause of the Constitution.

As part of the redistricting of state legislative districts triggered by the decennial census, the state legislature created a "majority-minority" district for the state senate in which black voters were in the majority. In creating this district, race was the predominant factor, because the intent was to favor black voters who have been historically disenfranchised in the state. A white voter in the district has filed a challenge to the creation of this district under the Equal Protection Clause. Seventy percent of the state's population is white.Is the court required to apply the strict scrutiny standard in examining the creation of this district? A Yes, because race was the predominant factor in creating the majority-minority district. B Yes, because the plaintiff is a racial minority in the majority-minority district. C No, because seventy percent of the state's population is white. D No, because the redistricting favors voters who have been historically disenfranchised in the state.

Answer choice A is correct. Under the Equal Protection Clause, election districts for public office may not be drawn using race as the predominant factor in determining the boundary lines, unless the district plan can survive strict scrutiny. Note that the district may be able to pass the strict scrutiny test if it can be shown that the district was drawn in compliance with federal Voting Rights Act, but the call of the question only asks whether strict scrutiny should apply. Answer choice B is incorrect because, while the plaintiff's status as a voter in the challenged district gives the plaintiff standing to bring this challenge, the fact that the plaintiff is a racial minority in the district does not require the court to apply strict scrutiny in determining whether the district violates the Equal Protection Clause. Answer choice C is incorrect because the fact that the plaintiff is not a racial minority in the state does not prevent the court from applying the strict scrutiny standard in determining whether the district violates the Equal Protection Clause. Answer choice D is incorrect because the strict scrutiny standard applies even when the district is drawn to favor historically disenfranchised groups if race was the predominant factor in creating the district.

The Employee Retirement Income Security Act (ERISA) provides that specific provisions of the Act "supersede any and all State laws insofar as they may now or hereafter relate to any employee benefit plan." Among those specific provisions is one that requires an employer with a group health plan to provide to each employee who would lose coverage as the result of termination of employment the opportunity to elect continuation coverage under the plan for a period of at least 60 days. Prior to the enactment of the continuation coverage election requirement, a state statute imposed a similar requirement on employers within the state with a group health plan.What is the effect of the federal provision on the state statute? A The federal provision preempts the state statute. B The state statute continues to be effective because it imposes the same requirements as the federal provision. C The state statute continues to be effective because it only applies to employers within the state. D The state statute continues to be effective because it was enacted before the federal provision.

Answer choice A is correct. Under the Supremacy Clause of Article VI, Section 2 of the U.S. Constitution, a federal law that expressly prohibits state regulation of a matter preempts state law on that matter. Here, ERISA provides that specific provisions of the Act, including the provision that addresses the right of an employee to elect continuation of health care coverage under an employer's group health plan, preempt state laws with regard to this right. Consequently, the state statute that provides for this right is expressly preempted. Answer choice B is incorrect because, even though the state law is consistent with the federal provision, the federal law expressly preempts the state law. Answer choice C is incorrect because federal law, as the "law of the land," applies to all states unless otherwise limited. The fact that the state law was limited to employers within the state does not protect it from preemption by a federal law that applies throughout the United States. Answer choice D is incorrect because the language of the federal law expressly preempts existing as well as future state regulation of this employee right. LEARN WHYNEXT QUESTION

A state healthcare act contained an ambiguous provision regarding the requirements for an insured person to bring a claim against an insurer. This provision in the state healthcare act was identical to one in the federal healthcare act, and relied in part on definitions contained in the federal act. An insurance company was sued under the state healthcare act. In its defense, the insurance company claimed that, based on the ambiguous provision, it could not be sued under the state act. When the case reached the state's highest court, that court sided with the insurance company, but it was unclear whether the court's decision relied on the court's interpretation of definitions contained in the federal act. The insured person petitioned the U.S. Supreme Court for review of the state court's decision.Is it constitutional for the Supreme Court to review the state court's decision? A Yes, because the Supreme Court may review any final judgment rendered by the highest court of a state. B Yes, because the Supreme Court may determine whether the state court decision relied upon a determination of a federal issue. C No, because the state court's decision could rest on adequate and independent state grounds. D No, because the suit by the insured person against the insurance company involved a state, not federal cause of action.

Answer choice B is correct. Although a final state-court judgment that rests on adequate and independent state grounds may not be reviewed by the U.S. Supreme Court, the Supreme Court may constitutionally review a state court decision to determine whether such grounds exist. Here, because it is unclear whether the state court opinion relied on the court's interpretation of definitions in the federal healthcare act, the Supreme Court may review the state court decision to make that determination. If the Supreme Court finds that the state court decision turns on a federal issue, then the Supreme Court may rule on whether the state court correctly decided the federal issue. If not, then the Supreme Court must dismiss the appeal. Answer choice A is incorrect because it overstates the power given to the Supreme Court by the Supremacy Clause. The Supreme Court may review a state court decision that involves a federal issue, but it may not review a state court decision that rests solely on adequate and independent state grounds. Answer choice C is incorrect. Although the state court decision could rest on adequate and independent state grounds, the Supreme Court may review the decision to determine whether it does. If it does, then the Supreme Court must dismiss the appeal. Answer choice D is incorrect. Even though a suit is based on a state cause of action, the Supreme Court may hear an appeal involving a federal issue that impacts the state cause of action.

Under a federal statute, a business had a valid, mature claim for compensation against a corporation, which the corporation refused to pay. The business properly assigned all rights to this claim to an assignee who agreed to litigate the claim in exchange for a fee and return any recovery received to the business. In the subsequent lawsuit filed by the assignee in federal court, the defendant corporation filed a motion to dismiss the action.Should the court grant this motion? A No, because standing is not required unless a constitutional right is violated. B No, because the assignee has standing to litigate this claim. C Yes, because the assignee has not suffered an injury in fact. D Yes, because any recovery would not redress an injury suffered by the assignee since the assignee is required to remit any recovery to the business.

Answer choice B is correct. An assignee has standing under Article III to bring an assigned claim, even if the claim is mature at the time of assignment and the assignment is made for collection only. Answer choice A is incorrect because standing relates to the power of the federal judiciary under Article III to hear a case. The case itself may involve rights arising from a federal statute as well from the Constitution. Answer choice C is incorrect because, although the assignor suffered the injury that gave rise to the claim, an assignee may sue on the basis of the assignor's injuries. Answer choice D is incorrect because the redressability requirement for standing is met when the court can redress the assignor's injury. LEARN WHYNEXT QUESTION

In general, the beef industry has been gravitating towards grass-fed cows instead of corn-fed cows, for reasons of health and taste. Grass fields that grow hay to feed grass-fed cows are currently located in only 10 states. In order to capitalize on this growing market as a source of revenue, Congress recently enacted a statute placing a high per-unit sales tax on the type of grass used for hay. The tax will discourage cow farmers from feeding their cows grass instead of corn because of the significant increase in the cost of grass over corn. Additionally, the increased cost will eventually be passed on to consumers, who analysts predict will be less inclined to choose expensive grass-fed beef over cheaper corn-fed beef.If the farmers who harvest and sell this grass file a lawsuit in federal court challenging the constitutionality of this sales tax, are they likely to succeed? A No, because Congress can enact any legislation that is necessary and proper to execute its authority to produce revenue. B No, because Congress has plenary power to impose and collect taxes if they are reasonably related to revenue production. C Yes, because application of the tax will not be apportioned properly between the states that have the necessary grass fields and those that do not. D Yes, because the tax violates the uniformity requirement of Article I, Section 8 of the Constitution.

Answer choice B is correct. Article I, Section 8 of the Constitution gives Congress the power to levy and collect taxes. A tax by Congress will generally be upheld if it has a reasonable relationship to revenue production, or if Congress has the power to regulate the activity being taxed. The sales tax on this variety of grass is reasonably related to revenue production, and there do not appear to be any provisions extraneous to tax purposes. Thus, the tax is likely constitutional. Answer choice A is incorrect because the Necessary and Proper Clause is not an independent source of power; it merely gives Congress the power to execute specifically granted powers. Thus, it cannot be the only enabling clause cited to justify the imposition of a tax. Answer choice C is incorrect because only direct taxes, such as ad valorem property taxes, are subject to the apportionment requirement. Answer choice D is incorrect because the requirement that taxes be uniform throughout the United States means only that the product must be identically taxed in every state in which it is found. Here, that requirement will be met.

Last year, Congress enacted legislation providing for funding opportunities to eligible secular and religiously affiliated colleges and universities. The funding will be available through individual counties as each county's funding limitations allow. The legislation does not require that each county apply standard guidelines nor does it provide any suggested guidelines other than a statement that "all counties should track funding and compile guidelines in the event of a federal audit."A county awards a large grant to a religiously affiliated college that employs a substantial number of residents. The grant contract, signed by representatives from both the county and the college, states as follows: "All grant monies must be used in compliance with county regulations. Further, the college must track the allocation of grant monies throughout the grant term."Is the county's award of the grant constitutional? A No, because the county provided grant monies to a religiously affiliated college. B No, because it does not require that the aid be used only for nonreligious purposes. C Yes, because the college is required to track funding. D Yes, because the college may be the subject of a federal audit.

Answer choice B is correct. Governmental financial assistance to religious institutions is permitted if the aid is secular in nature, used only for secular purposes, and, when the aid is distributed among secular and religious institutions, the distribution criteria must be religiously neutral. Answer choice A is incorrect because the government can provide grants to religiously affiliated colleges, provided the aid is secular in nature, used only for secular purposes, and the distribution criteria must be religiously neutral. Answer choice C is incorrect because tracking funding is insufficient; the government must explicitly require that funds not be used for religious purposes. Answer choice D is incorrect because even if a federal review of the award process does occur, the application of the regulations might still be unconstitutional. There is no indication that the county requires that the aid be spent for nonreligious purposes only.

A large utility company was located inside State A. The company burned coal to produce electricity, some of which was used within the state, but the majority of which was provided to neighboring states. In order to fund clean-up efforts made necessary as a result of burning coal, State A taxed the electricity the utility provided to its customers based on the amount used. Accordingly, even though some tax revenue came from electricity provided to State A residents, most of the tax revenue came from out-of-state residents. Residents of neighboring State B challenged State A's tax as unconstitutional, claiming that the tax disproportionately affected nonresident individuals.Assuming that Congress has not directly acted in the area, is the court most likely to find the tax constitutional or unconstitutional? A Constitutional, as a proper ad valorem tax. B Constitutional, as a proper state tax that comports with the Commerce Clause. C Unconstitutional, as a violation of the Privileges and Immunities Clause of Article IV. D Unconstitutional, as a violation of the Equal Protection Clause of the Fourteenth Amendment.

Answer choice B is correct. States may tax interstate commerce if Congress has not already acted in the particular area and if the tax does not discriminate against or unduly burden interstate commerce. The Supreme Court applies a four-part test to determine whether a state tax comports with the Commerce Clause: (1) there must be a substantial nexus between the activity taxed and the taxing state (here, that requirement is satisfied as the taxing state is both supplying the electricity and paying for clean-up), (2) the tax must be fairly apportioned according to a rational formula (here, tax is calculated by usage), (3) the tax may not provide a direct commercial advantage to local businesses over interstate competitors (nothing in the fact pattern indicates that this factor is present), and (4) there must be a fair relationship between the tax and the service provided (here, the tax is calculated in direct proportion to usage and is used for clean-up efforts related to production of electricity). Answer choice A is incorrect because it misstates the law; an ad valorem tax is based on the value of property located in the state. Answer choice C is incorrect because the tax does not discriminate against nonresident individuals; all individuals, whether in state or out of state, are subject to the same tax rate for the electricity provided. Answer choice D is incorrect because the tax is not discriminatory, and there is a rational basis to support the tax.

An environmentalist wanted to expose the amount of pollution being created by a power plant that used coal-burning power generators. To do this, the environmentalist broke into the power plant in order to steal the plant's monthly reports regarding the amount of carbon dioxide it released. The environmentalist successfully stole the reports. He then anonymously sent the report to the editor of a major environmental magazine, who used the stolen reports to write and publish a feature story about the amount of carbon dioxide pollution caused by the plant.If the plant files suit against the editor for the publication of illegally obtained private information, will it succeed? A No, because the editor, as a member of the press, has an unconditional right to publish information about matters of public concern. B No, because the editor played no part in unlawfully obtaining the monthly reports. C Yes, because the editor published private monthly reports not meant for public disclosure. D Yes, because the monthly reports were illegally obtained by the environmentalist.

Answer choice B is correct. The press has the right to publish information about matters of public concern, and the viewers have a right to receive it. The First Amendment shields the media from liability for publishing information that was obtained illegally by a third party as long as the information involves a matter of public concern and the publisher did not obtain it unlawfully. Here, the information regarding the amount of carbon dioxide pollution generated by the power plant is of great public concern. The environmentalist illegally obtained the monthly reports, but he sent them to the editor anonymously and not at the direction of the editor. Therefore, the editor is not liable for publishing the information. Answer choice A is incorrect because it is too broad. Although the press does have the right to publish information about matters of public concern, it cannot on its own illegally obtain information, and there may be governmental interests that warrant the prevention of publication. Answer choice C is incorrect because the press may have the right to publish private facts if they pertain to a matter of public concern. Answer choice D is incorrect because the First Amendment shields the media from liability for publishing information that was obtained illegally by a third party.

An environmental organization's stated mission is to support environmental causes. The organization's membership is generally open to the public, but its bylaws permit its officers to refuse to admit anyone to membership who does not adhere to the organization's mission statement.In a recent state administrative proceeding, the organization opposed plans to begin mining operations in the mountains surrounding a small town. Its opposition prevented the mine from being opened on schedule. In an effort to force the organization to withdraw its opposition, certain residents of the town attended a meeting of the organization and tried to become members, but the officers refused to admit them. The residents sued the organization, claiming that the refusal to admit them was discriminatory and violated a local ordinance that prohibits any organization from discriminating on the basis of an individual's political views. The organization responded that the ordinance is unconstitutional as applied to its membership decisions.Are the residents likely to prevail in their claim? A No, because the membership policies of a private organization are not state action. B No, because the organization's right to freedom of association allows it to refuse to admit potential members who do not adhere to its mission statement. C Yes, because the action of the officers in refusing to admit the residents as members violates equal protection of the laws. D Yes, because the ordinance serves the compelling interest of protecting the residents' free speech rights.

Answer choice B is correct. The residents are not likely to prevail in their claim, because it would violate the environmental organization's First Amendment right to freedom of association if the state were to force the organization to accept the residents as members. The U.S. Supreme Court has held that the forced inclusion of an unwanted person in a group violates the group's freedom of association if including that person would significantly affect the group's ability to express its viewpoints. The freedom of association entitles the environmental organization to refuse membership to the residents, because admitting them would effect a change in the organization's viewpoint on the mining operations. Answer choice A is incorrect because, while it is true that the membership policies of a private organization are not state action, the local ordinance on which the residents base their suit is state action, and it is subject to the requirements of the First Amendment. Answer choice C is incorrect because the action of the officers in refusing to admit the residents as members is not subject to the equal protection clause. The environmental organization is a private entity, and therefore the conduct of the organization's officers does not constitute state action. Answer choice D is incorrect because the U.S. Supreme Court has held that even statutes that support compelling interests do not justify the severe burden on an organization's freedom of association that would result from forcing an organization to accept members who would significantly affect the organization's ability to express its viewpoints.

A state enacted a law that prohibited the sale of violent video games to minors and imposed a fine for each violation. The legislative history demonstrated a concern that there was a correlation between playing such games and subsequent violent behavior. A maker of video games brought suit contending that this law violated its First Amendment right of free speech.Is this law unconstitutional? A Yes, because the costs of such a restriction on speech outweigh its benefits. B Yes, because the state law is a content-based restriction. C No, because video games do not qualify for First Amendment protection. D No, because states have the power to protect children from harm.

Answer choice B is correct. The state law prohibited a particular type of speech (i.e., violent video games), and thus contained a content-based restriction subject to strict scrutiny. Such regulations must be necessary to achieve a compelling government interest and narrowly tailored to meet that interest. A mere correlation between violent video games and violent behavior does not constitute a compelling interest to regulate such games. Accordingly, the law is unconstitutional. Answer choice A is incorrect because a balancing test is not the correct test to determine the constitutionality of content-based regulations. Rather, content-based regulations are subject to the strict scrutiny test. Answer choice C is incorrect because video games communicate ideas, and are thus a form of speech protected by the First Amendment. Answer choice D is incorrect because, although a state possesses the power to protect children from harm, a state may not shield children from speech absent a recognized limitation on the speech, such as obscenity.

A state raised its minimum wage to $15 per hour. A federal post office in the state pays some of its employees the federal minimum wage of $7.25 an hour.Can the post office be prosecuted for failing to adhere to the new state minimum wage? A No, because the state minimum wage is preempted by federal law. B No, because the state cannot regulate the federal government absent congressional consent. C Yes, because the state minimum wage is not a tax on the federal government. D Yes, because the post office at which the federal employees are employed is located in the state.

Answer choice B is correct. The states have no power to regulate the federal government unless Congress permits the state regulation or unless the state regulation is not inconsistent with existing federal policy. Here, the state's minimum wage statute penalizes the behavior of an agency of the federal government, which is immune from that regulation under the concept of sovereign immunity. Answer choice A is incorrect. When federal and state governments legislate in the same area, the Supremacy Clause provides that federal law supersedes conflicting state law. However, there is no conflict here that will trigger preemption. A state law conflicts with federal law if it allows conduct that is forbidden by the federal law or makes it impossible (or nearly so) to comply with both. Here, an employer can easily comply with both the state and federal law when it obeys the state law. Therefore, preemption does not apply. Answer choice C is incorrect because the states have no power to regulate the federal government unless Congress permits the state regulation or unless the state regulation is not inconsistent with existing federal policy. This immunity is not limited to state taxation, and even some state taxation may be permitted. Answer choice D is incorrect. Even though the post office employees are working within the state, the employees are working for an agency of the federal government. Although the employees' wages can be subject to state income tax, the federal government as an employer is not subject to the state's minimum wage.

A city passed an ordinance prohibiting all "adult entertainment establishments," defined as "enterprises that sell, trade, or depict materials that are obscene or pornographic." The city justified its law on the basis of reputable studies showing that (i) obscene and pornographic material degrades females and increases the tendency towards anti-social behavior of people who view it; and (ii) adult entertainment establishments are linked with criminal activity such as prostitution and drug dealing. The city also emphasized that the three cities bordering it all allow adult entertainment establishments. An entrepreneur who wishes to open an adult bookstore in the city sues the city in an appropriate federal court and claims that the ordinance is unconstitutional. Will she likely prevail? A Yes, because the city's ordinance deprives her of her right to earn a living in violation of the Due Process Clause of the Fourteenth Amendment. B Yes, because the First Amendment prohibits the city from banning all adult entertainment establishments. C No, because the plaintiff has not been deprived of her constitutional right to earn a living, as she can open her adult bookstore in one of the three nearby cities. D No, because the city has legitimate reasons for banning adult entertainment establishments that are not based on the content of the material they are selling, trading, or depicting.

Answer choice B is correct. Under the First Amendment, a city can regulate adult entertainment establishments (e.g., by localizing them in a particular area), but cannot impose a blanket ban on all pornography. It does not matter that (i) the city might have legitimate reasons for such a ban (e.g., decreasing crime), or (ii) other neighboring cities allow such establishments. For these reasons, answer choices C and D are incorrect. Answer choice A is incorrect because the substantive aspect of the Due Process Clause does not impose a strict scrutiny standard on governmental interference with economic rights, such as the right to earn a living. Such interference instead is judged under the rational basis standard, which the ordinance in question satisfies.

The owner of a high rise building in a large metropolis that extends over two states applied for a permit to construct a helicopter pad on the roof of the building to permit building tenants to use a heliocopter commuting service. The local zoning board in the state where the building is located denied the permit based on existing zoning regulations. Prior to seeking a special use variance, the owner filed suit in federal court seeking a declaratory judgment that the zoning board's denial violated a federal law that prohibits a state or local government from effectively denying the provision of interstate transportation services to the public.Of the following, which is likely the local government's best constitutional defense against this suit? A The building owner does not have standing; only the building's occupants could challenge the zoning based on violation of this ordinance. B The matter is not ripe for adjudication. C The suit is barred by the Eleventh Amendment. D The suit violates the "case or controversy" requirement of Article III, Section 2 of the Constitution because it seeks a declaratory judgment.

Answer choice B is correct. Under the ripeness doctrine, which is grounded in part on the "case or controversy" requirement of Article III, Section 2, a federal court will not consider a claim before it has fully developed, as doing so would be premature, and any potential injury would be speculative. Here, because the building owner has failed to seek a special use variance, it is likely that the court will find the building owner's claim is premature. Answer choice A is incorrect. The building owner would suffer an injury in fact without the zoning variance. Answer choice C is incorrect. The Eleventh Amendment applies only to states and state agencies. Local governments (e.g., counties, cities) are not immune from suit. Answer choice D is incorrect because the fact that a plaintiff is seeking a declaratory judgment as a form of relief does not violate the "case or controversy" requirement.

Congress has passed legislation authorizing an appropriation of $5 million to buy land adjacent to a national historic battlefield in order to prevent its commercial development.Of the following, which provides the best constitutional justification for this congressional action? A Takings Clause of the Fifth Amendment B Spending power found in Article I C Property power found in Article IV, Section 3 D Obligations of Contracts Clause of Article I, Section 10

Answer choice B is correct. Under the spending power found in Article I, Section 8 of the U.S. Constitution, Congress has the power to spend for the "general welfare"—i.e., any public purpose—not just to pursue its other enumerated powers. The public purpose need not be a national purpose, but may instead concern only a particular locale. Therefore, this appropriation of funds in order to acquire land is likely to be permitted by the congressional spending power. Answer choice A is incorrect because the Takings Clause of the Fifth Amendment is only a restriction on the power of the federal government (and through the Fourteenth Amendment, state governments) to take private property without just compensation. It is not a source of legislative power for Congress. Answer choice C is incorrect. Under the property power found in Article IV, Section 3 of the U.S. Constitution, Congress has the power to act as a proprietor as well as a legislature with regard to federal lands. This includes the power to dispose of federal land as Congress sees fit, but does not authorize the acquisition of such lands. Answer choice D is incorrect because Article I, Section 10 prohibits only a state legislature from passing any law "impairing the obligation of contracts." This prohibition does not apply to Congress.

A federal law provides that the President may request that Congress rescind an appropriation of funds. The law also provides that, if both the House and Senate have not approved legislation rescinding the appropriation within 45 days of the request, the President is required to expend the funds.The President, finding that specific congressional legislation mandating the expenditure of funds to repair a federal facility that was slated for closure was unnecessary, vetoed the legislation. Congress passed the legislation over the President's veto by a two-thirds vote in both the Senate and the House. The President then requested that Congress rescind the appropriation. Two months after the request, neither the House nor the Senate had taken any action on the President's request.Is the President constitutionally required to expend funds for the repair of the federal facility? A Yes, because congressional passage of the legislation over the President's veto prevents the President from exercising his power to impound funds. B Yes, because Congress did not act on the President's request. C No, because the President vetoed the original appropriations legislation. D No, because the President determined that the expenditure of funds was unnecessary.

Answer choice B is correct. When Congress explicitly mandates an expenditure of funds, the President has no constitutional power to impound those funds (i.e., refuse to spend them or delay the spending). Consequently, although Congress has provided a statutory means by which the President can request that Congress rescind an appropriation of funds, because Congress did not act on that request, the President is constitutionally required to expend the congressionally mandated funds. Answer choice A is incorrect because the President's lack of power to impound funds when Congress has mandated their expenditure does not turn on whether the authorizing legislation has been passed over the President's veto. The President lacks this power whenever Congress mandates that the funds be expended. Answer choice C is incorrect because, although the President can constitutionally veto legislation, the legislation can become law if Congress passes the legislation over the President's veto. Here, Congress overrode the President's veto by passing the legislation by a two-thirds vote in both the Senate and the House. Answer choice D is incorrect because, while the President determined that the expenditure of the funds was unnecessary, the President cannot refuse to expend funds appropriated by Congress when Congress mandates the expenditure of those funds, as Congress did in this instance.

A federal statute provides that a U.S. territory can freely trade certain agricultural products, including bananas, with a foreign island nation. Shortly after the enactment of this statute, a U.S. state enacted a law allowing for a free transfer of goods between the state, the territory, and the island nation. Several years later, the President of the United States entered into a self-executing treaty with several trading partners that in pertinent part required a mark of origin on all agricultural products from all foreign countries. The Senate promptly ratified this treaty. Last year, the President of the United States entered into an executive agreement with the leader of a second foreign country, prohibiting the importation into the United States of bananas from any other country. Citizens of the U.S. territory want to continue to freely buy bananas from the island nation.Which of the following is likely to govern whether citizens of the U.S. territory can buy bananas from the island nation? A The executive agreement B The federal statute passed by Congress C The federal treaty D The state law

Answer choice B is correct. When a federal statute conflicts with an executive agreement, the federal statute takes precedence over the executive agreement. Here, the federal statute and the executive agreement directly conflict with one another, so the federal statute governs whether the territory can buy bananas from the island nation. Answer choice A is incorrect because, even though the President does have the power to enter into executive agreements with foreign nations, the executive agreement is subject to federal statutes. Answer choice C is incorrect because a federal statute and a federal treaty are roughly equivalent. When the two conflict, whichever was enacted more recently would govern. However, here, the two do not conflict. The federal treaty does not govern whether the territory can buy bananas from the island nation; it only requires that a mark of origin be included on those bananas. Answer choice D is incorrect because federal actions, whether statutes, treaties, or executive agreements, take precedence over state laws.

A woman obtained a restraining order that required her estranged husband to stay away from her and their children. Subsequently, the woman informed local police officers several times during the course of an evening that her husband had violated this order by seizing the children. The police officers, acting in accord with their statutory discretion, took no action to locate the husband or the children. Later that night, the husband killed the children. The woman filed an action in federal court alleging that the police officers' failure to enforce the restraining order violated her procedural due process rights under the Fourteenth Amendment.Of the following, which provides the best defense to this action? A Persons who are protected by a restraining order do not constitute a suspect or quasi-suspect class. B Providing notice and a hearing is an inappropriate response with regard to a request to enforce a restraining order. C The enforcement of the restraining order was subject to the discretion of the police officers. D The right to enforcement of a restraining order does not constitute a fundamental right.

Answer choice C is correct. A cognizable property interest involves more than an abstract need or desire; there must be a "legitimate claim of entitlement" by virtue of statute, employment contract, or custom. If enforcement of the restraining order is subject to the discretion of the police officers, then the woman cannot assert an absolute entitlement to having the order enforced. Unless the woman was entitled to have the restraining order enforced, she did not have a protected interest that triggered her procedural due process rights. Answer choice A is incorrect. The existence of a suspect or quasi-suspect class is relevant in determining the level of scrutiny to which governmental conduct is subjected when a violation of the Equal Protection Clause is at issue. Although persons who are protected by a restraining order do not constitute a suspect or quasi-suspect class, this fact is irrelevant in determining whether the police officers' failure to enforce the restraining order constituted a procedural due process violation. Answer choice B is incorrect. Although typically procedural due process requires both notice and a hearing, neither is necessarily required. More importantly, the process required does not become an issue if the party does not possess a protected interest under the Due Process Clause. Answer choice D is incorrect. Whether a right is a fundamental right affects the level of scrutiny that is applicable when governmental conduct is challenged as a violation of substantive due process rights. On the other hand, to raise a procedural due process claim, the right to enforcement of a restraining order must merely create a protected liberty or property interest. Therefore, the argument that the right to enforcement of a restraining order does not constitute a fundamental right would be ineffective against the woman's claim.

For the past ten years, a city has maintained an internet website. Pursuant to a rule adopted by the city council, anyone, including public employees and officials in addition to the general public, was permitted to post comments on the city's website. The webmaster was tasked with reviewing the posted comments and removing any that were obscene or irrelevant. After the website was recently inundated with comments regarding a particular ordinance passed by the council, the council ordered the webmaster to remove the feature from the city's website that permitted the posting of comments. A resident of the city has filed a suit challenging the city's action as a violation of the First Amendment.Of the following, which provides the best support for the resident's argument that the comments section of the city's website constitutes a public forum? A Because the website has existed for at least 10 years, it qualifies as a traditional public forum. B Public employees and officials could post comments on the website. C The city permitted the general public to post comments on the website. D The website is freely accessible by anyone in the general public.

Answer choice C is correct. A public forum is a place that has been devoted to public discourse either traditionally or by governmental designation. In this case, because the city has permitted the public to post comments on its website, it is likely that the court would find that the website constitutes a public forum. Answer choice A is incorrect because a public forum can take two forms—a traditional public forum or a designated public forum. Here, although the comments section of the city's website may constitute a designated public forum, its existence for the past ten years is highly unlikely to be a sufficient length to qualify the website as a traditional public forum. Answer choice B is incorrect because comments made by public employees and officials in their role as such would constitute governmental speech for which the city is not subject to First Amendment constraints. Answer choice D is incorrect. Although the public must be able to access a place in order for it to be a public forum, access alone is not sufficient to establish a place as a public forum. For example, places such as airport terminals to which the public has access are not considered to be public forums. LEARN WHYNEXT QUESTION

A state prohibits voting for a write-in candidate in both primary and general elections. However, the state provides a mechanism by which individuals who want to appear as candidates on the primary ballot, as well as groups that want to be classified as a party, can do so.Of the following, which is the best argument that a voter challenging the constitutionality of this ban on write-in candidates can make? A The right to vote is a fundamental right, so the state ban is subject to strict scrutiny review. B The state ban imposes a burden on the right of voters to select the candidates of their choice. C The mechanism provided by the state does not provide for reasonable access to the ballot. D The Fifteenth Amendment limits a state's power to regulate its own elections.

Answer choice C is correct. A state may ban all write-in candidates in both primary and general elections, at least when the state provides reasonable means by which a candidate can get on the ballot. Consequently, the best argument for challenging this ban is that the state fails to provide reasonable means by which a candidate can get on the ballot. Answer choice A is incorrect because, while the right to vote is a fundamental right, governmental restrictions on this right are not necessarily subject to strict scrutiny review. A state may ban all write-in candidates in both primary and general elections, at least when the state provides reasonable means by which a candidate can get on the ballot. Answer choice B is incorrect because, even though a state ban on write-in candidates does burden the right of voters to select candidates of their choice, this ban is not unconstitutional provided the state provides reasonable means by which a candidate can get on the ballot. Answer choice D is incorrect because, while the Fifteenth Amendment does limit the power of states to control their own elections, this amendment is limited to the denial of the right to vote by a state (or the federal government) on the basis of race, color, or previous condition of servitude. The denial of all write-in candidates, at least on its face, does not run afoul of this prohibition. Therefore, this rule is not the best argument for challenging this ban.

An organization against drunk driving sought permission from the owner of a mall to pass out leaflets in favor of tougher drunk driving laws in front of a liquor store. The mall owner denied the organization permission. The organization, filing an action in an appropriate court, sought an injunction permitting the organization to pass out its leaflets in accord with its free speech rights. The state's highest court, interpreting the state constitution, permits the exercise of free speech rights on private property that is regularly held open to the public. Of the following reasons, which is the best argument for granting the injunction? A Leafleting is a form of speech that is protected by the First Amendment of the United States Constitution. B The reason for leafleting was related to the place where the organization sought to leaflet. C The leafleting was permitted by the state constitution. D By admitting members of the general public, the mall constituted a limited public forum.

Answer choice C is correct. Although, for the reasons discussed below, the First Amendment to the United State Constitution does not require the mall owner to permit leafleting in this instance, the state constitution as interpreted by the state's highest court does. A state may expand, but cannot contract, individual rights, such as free speech, granted by the United States Constitution. Answer choice A is incorrect because, although leafleting is a form of speech that enjoys First Amendment protection, such protection does not extend to leafleting on private property, even private property that is open to the public such as a mall. Answer choice B is incorrect because the fact that the purpose for leafleting has a logical relationship to the location where the leafleting is to be conducted does not trigger First Amendment protection of the leafleting, where that location is private property. Answer choice D is incorrect because, as noted with respect to answer choice A, the opening of private property to the public does not convert it into a public or limited public forum at which leafleting must be permitted despite the objection of the owner of the property.

A state statute provides for a reduction in the property tax assessed against any residential dwelling located in a particular economically depressed area of the largest city in the state. The purpose of the statute is to encourage homeownership in the designated area and thereby improve the economic wellbeing of the residents living there. The statute provides that, in order to qualify for this tax break, the homeowner must be a citizen of the United States.Is the statute likely constitutional? A Yes, because the state can offer tax breaks to specific citizens under the Tenth Amendment. B Yes, because the state has a rational basis for providing for the tax break to specific citizens. C No, because the tax break offered to specific citizens by the state violates the Equal Protection Clause. D No, because the state cannot legislate with regard to alienage.

Answer choice C is correct. Generally, a state law that discriminates against aliens is subject to strict scrutiny under the Equal Protection Clause and will be struck down. While a state law that restricts or prohibits an alien's participation in government functions is subject only to a rational basis review, this statute does not fall within that category. For this reason, answer choice B is incorrect. Answer choice A is incorrect because, while the Tenth Amendment reserves to the states powers not delegated to the federal government nor prohibited by the constitution, the limitation of a tax break to citizens of the United States violates the Equal Protection Clause. Answer choice B is incorrect. Although the state had a rational basis for enacting the statute, the rational basis standard only applies when a state law prohibits an alien's participation in government functions. As discussed above, this statute is subject to strict scrutiny because it discriminates against aliens regarding the tax break in violation of the Equal Protection Clause. Answer choice D is incorrect because a state may legislate with regard to alienage if Congress has explicitly authorized it.

The association that regulated all state-licensed realtors organized its members into 20 regions. Four representatives were chosen in each region to join a board to advocate for the rights of all realtors in that area. The board positions were unpaid, and the association established a rule requiring that two of the four chosen representatives must be black. The stated goal of this rule was to address the history of societal discrimination against people of color in the state. In general, there are fewer black realtors in the state than white realtors.If no federal statute applies, is this rule constitutional? A Yes, because the rule is rationally related to a legitimate governmental interest. B Yes, because the rule is intended to help remedy past race-based discrimination. C No, because the rule only remedies a general societal injustice. D No, because the rule is not substantially related to an important governmental interest.

Answer choice C is correct. Programs that discriminate based on race, even if they favor racial or ethnic minorities, are subject to strict scrutiny; and for a governmental affirmative action program based on race to survive, the relevant governmental entity must show more than a history of societal discrimination. The government—whether federal, state, or local—must itself be guilty of specific past discrimination against the group it is seeking to favor, and the remedy must be narrowly tailored to end that discrimination and eliminate its effects. Here, the state board's attempt to remedy general societal racial discrimination, rather than a specific practice of discrimination, does not meet this requirement. Answer choice A is incorrect because it improperly applies the rational basis test to this rule; strict scrutiny is warranted. Answer choice B is incorrect because, to survive, a governmental affirmative action program must remedy specific past discrimination carried out by the government rather than a general societal injustice. Answer choice D is incorrect because it applies the test for intermediate scrutiny where strict scrutiny should apply.

A state law prohibited all outdoor advertisement of cigars within 1,000 feet of a school. The legislative history for the statute demonstrates that the prohibition was intended to combat cigar use by minors, which legislators believed was a pressing problem. Federal law does not prohibit state regulation of cigars. Moreover, in order to receive federal funding for state substance abuse programs, a state must prohibit the sale of tobacco products to minors.Cigar manufacturers, distributors and retailers have challenged the state law as unconstitutional. They note that the state prohibition on outdoor advertisement encompasses ads that are neither untruthful nor misleading. In addition, they point out that the law is not narrowly tailored to protect students while also permitting legal advertisement of cigars to adults because in certain cities, the law prevents outdoor advertising of cigars in almost all areas of the city.Is the court likely to strike down the law? A No, because the state's interest in the health of minors is compelling and dovetails with the federal purpose of preventing minors from smoking. B No, because the law directly advances the state's compelling interest. C Yes, because the law is not narrowly tailored to achieve its purpose. D Yes, because the law prohibits truthful speech.

Answer choice C is correct. Restrictions on commercial speech, such as advertising, are subject to intermediate scrutiny and are reviewed under a four-part test: (i) it must concern lawful activity and be neither false nor misleading; (ii) the asserted government interest must be substantial; (iii) the regulation must directly advance the asserted interest; and (iv) the regulation must be narrowly tailored to serve that interest. Although the state's concern for the health of minors constitutes a compelling interest, the law is not narrowly tailored to serve that interest. Thus, the state law constitutes an unconstitutional restriction on commercial speech. Answer choice A is incorrect because the regulation is an impermissible restriction on commercial speech because it is not narrowly tailored. Answer choice B is incorrect because, even though the law directly advances the state's substantial purpose, it is not narrowly tailored to serve that purpose. Answer choice D is incorrect because a state may ban truthful commercial speech if there is a substantial government interest and the regulation directly advances that interest and is narrowly tailored to serve that interest. LEARN WHYNEXT QUESTION

A federal statute generally makes age discrimination in the hiring or firing of employees illegal and provides for a civil action for damages against the offending employer. The statute applies to public as well as private employers and contains a clear statement of Congressional intent to abrogate state immunity. In federal court, an employee sued her employer, a state agency, for violation of this statute and sought relief in the form of retroactive money damages. The state agency moved to dismiss the action as constitutionally prohibited. The state employee conceded that the age discrimination that she experienced was rationally related to a legitimate state interest.Should the court dismiss the action on constitutional grounds? A No, because Congress was acting pursuant to its power under the Commerce Clause. B No, because Congress was acting pursuant to its power under the Enabling Clause of the Fourteenth Amendment. C Yes, because the Eleventh Amendment prevents the recovery of retroactive money damages by citizens against a state agency. D Yes, because the strict scrutiny test must be met for there to be a violation of the Equal Protection Clause.

Answer choice C is correct. The Court has interpreted the Eleventh Amendment as barring unconsented private suits against a state for retroactive money damages. Congress may abrogate state immunity from liability if it is clearly acting to enforce rights created by the remedial provisions of the Thirteenth, Fourteenth, and Fifteenth Amendments (i.e., the Civil War Amendments), and does so expressly. Here, Congress is not remedying conduct that violates the Fourteenth Amendment's Equal Protection Clause, as age is not a suspect or quasi-suspect class and thus a state's age-based discrimination merely needs a rational basis. That is why answer choice B is incorrect. Answer choice A is incorrect because Congress cannot abrogate state sovereign immunity under the Commerce Clause. Answer choice D is incorrect because the Equal Protection Clause is not implicated. The federal statute at issue applies to all employers equally, whether public or private. Regardless, answer choice D is a misstatement of the law because rational basis review applies in equal protection cases that do not involve a suspect or quasi-suspect class.

A state statute created a governmental assistance program to provide single mothers with funds to pay for childcare and early childhood education so the mothers can work before their children start public school. However, the program is only available to United States citizens. A single mother, who is a resident alien in the state, applied for governmental assistance under this program and was denied. She has challenged the constitutionality of the state program's restriction to United States citizens.Is the program likely to be found unconstitutional? A No, because an alienage classification is likely valid unless it is arbitrary and unreasonable. B No, because the program does not discriminate based on state citizenship. C Yes, because strict scrutiny applies to state laws that discriminate against aliens. D Yes, because the program violates the Privileges and Immunities Clause of Article IV.

Answer choice C is correct. The Court will generally apply the strict scrutiny test and strike down state laws that discriminate against aliens, such as laws prohibiting aliens from owning land, obtaining commercial fishing licenses, or being eligible for welfare benefits or civil service jobs. Therefore, this program will probably be found to be unconstitutional under strict scrutiny review. Answer choice A is incorrect because this standard only applies to federal alienage classifications. Answer choice B is incorrect. Although a state may not discriminate against citizens of another state under the Comity Clause, a state also may not discriminate against aliens unless the strict scrutiny test is satisfied. Here, the state statute discriminates against aliens. Answer choice D is incorrect because the Privileges and Immunities Clause of Article IV, also known as the Comity Clause, prohibits one state from discriminating against the citizens of another state. In this context, the term "citizen" does not include corporations or aliens. Therefore, it does not apply to this state program. LEARN WHYNEXT QUESTION

Article I, § 4 of the Constitution provides: "The times, places and manner of holding elections for Senators and Representatives shall be prescribed by each state legislature, but Congress may . . . make or alter such regulations." Congress enacted a statute requiring every state to allow voters to register to vote in federal elections either by mail or at a state motor vehicle department. If a state refuses to comply with the statute and is sued by the federal government, will the state likely prevail? A Yes, because Congress cannot "commandeer" state legislatures to enact statutes. B Yes, because Congress cannot "commandeer" state executive officials to carry out federal programs. C No, because Article I, § 4 permits Congress to require states to change their laws regarding federal elections. D No, because the statute, which applies to federal elections only, does not interfere with a traditional government function.

Answer choice C is correct. The Elections Clause of Art. I explicitly empowers Congress to override state laws concerning federal elections. This express provision makes irrelevant general principles of federalism embodied in the "commandeering" cases. For this reason, answer choices A and B are incorrect. Answer choice D is incorrect because, although conducting elections is a traditional function of state governments, the Elections Clause allows congressional interference with that function to the extent that it involves the election of United States senators and representatives.

Due to the scarcity of post offices in low-income inner-city communities, the few post offices in the communities suffered severe overcrowding, slowing postal service to and from these communities significantly. To address the problem, Congress enacted a statute authorizing the conversion of old, uninhabited buildings in these communities that were owned by the government into post offices. A historical society in one of the affected communities wants to stop Congress from converting the buildings into post offices because they were categorized as historical buildings of cultural value to the community.Is the statute constitutional? A No, because establishing postal services is a power reserved to the states. B No, because the statute violates the Takings Clause. C Yes, under Congress's property power. D Yes, because the properties will be converted for a public use.

Answer choice C is correct. The Federal Property Clause of Article IV, Section 3 gives Congress the "power to dispose of and make all needful rules and regulations respecting the territory or other property belonging to the United States." In addition, there is no express limit on Congress's power to dispose of property owned by the United States. Here, Congress has the power to make all needful rules respecting the old buildings owned by the government, including converting them into post offices. Answer choice A is incorrect because it misstates the law. Congress, not the states, has the exclusive power "to establish post offices and post roads" under Article I, Section 8, Clause 7. Answer choice B is incorrect because the Takings Clause does not apply when the government is putting property owned by the government to a use that does not rise to the level of a regulatory taking of surrounding property. Answer choice D is incorrect because there is no limitation under Article IV, Section 3, that Congress can only use properties owned by the government for a public use.

A state board of transportation ordered a railroad company to sell a parcel of land adjoining its railroad track. The parcel in question had been part of a much larger section of land transferred a number of years before from the state to the railroad company in exchange for the company's provision of railroad services to the citizens of the state. The state board fixed a reasonable price based on the land's fair market value to compensate the railroad company for the loss of its land. The railroad refused to sell the land to the designated buyer, a farmer's cooperative. The private cooperative planned to build a warehouse on the land in order to store its members' produce for shipment by rail and other means.In an action to compel the railroad to comply with the order of the state board, should the court rule in favor of the state board? A Yes, because the railroad is regulated by the state board of transportation. B Yes, because the price set by the board constitutes just compensation. C No, because the order violates the Fourteenth Amendment's incorporation of the Fifth Amendment. D No, because the order originated with a state board rather than a state legislature.

Answer choice C is correct. The Fifth Amendment Takings Clause, which applies to the states through the Fourteenth Amendment, provides that a government may seize private property not only for its own direct use, but also in order to transfer the property to another private party. Such a seizure is permissible if it is rationally related to a conceivable public purpose. Under these facts, the land was intended to be used by a farmer's cooperative for building a warehouse and storing its produce. It is unlikely that the intended use of the land was rationally related to a conceivable public purpose since the facts do not indicate that the seizure was based on economic redevelopment goals or safety and welfare justifications. Answer choice A is incorrect because, although the transfer of private property to a regulated public utility might be justified under the Takings Clause as satisfying the public purpose requirement, it cannot be forced to sell its land to another private person unless there is a public purpose that justifies the transfer. Here, it is unclear that there is a sufficient public purpose for taking the land. Answer choice B is incorrect because, although a private-to-public transfer of property can be constitutional when just compensation is paid to the private property owner, a prohibited private-to-private transfer is not rescued by the payment of just compensation. Answer choice D is incorrect because the Takings Clause is not limited to legislative action but applies to actions taken by other state entities, such as the transportation board in question, as well. LEARN WHYNEXT QUESTION

Relying on its Commerce Clause power, Congress enacted a statute that criminalized the knowing possession of sexually explicit visual displays of a minor. Congress adduced facts that possession of such displays substantially affected interstate commerce. Simulated displays, or those that use youthful looking adults or computer graphic techniques instead of minors, are not prohibited. Amalgamate displays, or those that contain actual minors but with the sexually explicit aspect created by using youthful looking adults or computer graphic techniques, are prohibited. The definition of the crime is neither vague nor overbroad.A defendant who possessed amalgamated displays in his home and did not sell or otherwise transfer them was convicted under this statute. He has challenged his conviction, contending that the statute is unconstitutional.Should an appellate court uphold his conviction? A No, because the statute exceeds congressional power under the Commerce Clause by regulating the possession of material that has not entered into the stream of commerce. B No, because the statute violates the defendant's privacy interest which is protected by the Fifth Amendment Due Process Clause. C Yes, because the statute does not violate the First Amendment Free Speech Clause. D Yes, because there is no constitutional right to possess pornography.

Answer choice C is correct. The First Amendment does not protect child pornography. Because of the government's compelling interest in protecting minor children from exploitation, the sale, distribution, and even private possession of child pornography may be prohibited, even if the material would not be obscene if it involved adults. Simulated displays may not be banned, but amalgamated displays may be banned. Accordingly, the statute passes constitutional muster. Answer choice A is incorrect because Congress has broad power under the Commerce Clause to regulate interstate commerce. This power extends to privately produced and consumed items when those items have a substantial impact on interstate commerce, provided that Congress has adduced facts that establish this impact. Answer choice B is incorrect because Congress may criminalize the possession of child pornography without violating an individual's due process rights. Answer choice D is incorrect because there is a fundamental right to possess obscene materials, including pornography that does not include children, in the privacy of one's home.

A member of the federal House of Representatives was participating in a committee hearing regarding tax policies for tobacco. The member was in favor of lowering the federal excise tax on tobacco products, and stated his opinions in his own committee report recommending a reduction in the excise tax. It was later discovered that the member had solicited and accepted a bribe from a large tobacco producer if he agreed to recommend the reduction in the federal excise tax.If the member is prosecuted for these actions, can the member invoke the Constitution to receive immunity from the charges? A Yes, because the Speech or Debate Clause protects members of Congress from liability for statements and conduct made in the regular course of the legislative process. B The member may assert absolute legislative immunity from civil liability during his term in office, but he may still be criminally liable for soliciting and accepting a bribe. C No, because the Speech or Debate clause does not foreclose prosecution for a crime when the crime does not require proof of legislative acts. D No, because the Speech or Debate Clause would only protect the member from liability for speech made on the floor of Congress, not comments made in a committee report.

Answer choice C is correct. The Speech or Debate Clause of Article I, Section 6 protects members of Congress from civil and criminal liability for statements and conduct made in the regular course of the legislative process, including a speech given on the floor of Congress, committee hearings, and reports. However, this protection does not foreclose prosecution for a crime, including the taking of bribes, when the crime does not require proof of legislative acts or inquiring into the motive behind those acts. Here, the member solicited and accepted a bribe regarding the lowering of a federal excise tax on tobacco products. This crime does not require proof of a legislative act or the motive behind the member's report, so he is subject to criminal liability. Answer choice A is incorrect because this protection will not grant the member immunity for taking a bribe to make his recommendation. Answer choice B is incorrect because there is no such thing as absolute legislative immunity during a term in office. Answer choice D is incorrect because it misstates the legal effect of the Speech or Debate Clause, which can grant immunity for statements and conduct made in the regular course of the legislative process, including those made in reports.

A proposed federal statute would make it a crime to force anyone to engage in prostitution against his or her will anywhere in the United States.Is this proposed statute likely unconstitutional? A Yes, under the Tenth Amendment, because it invades a traditional area of state regulation. B Yes, because Congress cannot enact criminal laws. C No, because the Thirteenth Amendment gives Congress the power to eliminate involuntary servitude. D No, because the Necessary and Proper Clause allows Congress to legislate for the general welfare.

Answer choice C is correct. The Thirteenth Amendment gives Congress the power to adopt legislation rationally related to eliminating involuntary servitude. This power has been broadly interpreted to allow Congress to regulate both private and government action and is the only Amendment that authorizes Congress to regulate purely private conduct. Therefore, the proposed statute would likely be constitutional as it addresses forcing anyone to work against his or her will. Answer choice A is incorrect because the Tenth Amendment reserves to the states only the powers that are not specifically given to the federal government. The power to eliminate involuntary servitude has been specifically delegated to Congress by the Thirteenth Amendment. Answer choice B is incorrect. Although Congress cannot pass bills of attainder (i.e., legislative acts that criminally punishes some person or group without a trial) or enact ex post facto laws (i.e., a retroactive change to a criminal or penal law), there is no general prohibition keeping Congress from enacting criminal laws. Answer choice D is incorrect because the Necessary and Proper Clause gives Congress the power to enact any legislation necessary and proper to execute any authority granted to any branch of the federal government, but this clause is not an independent source of power.

Animal rights advocates put pressure on Congress to address the mistreatment of animals used in scientific research. In response, Congress passed legislation conditioning the receipt of federal research grants on adhering to specific standards in the treatment of laboratory animals.Which of the following constitutional provisions most strongly supports the enactment of this legislation? A The Necessary and Proper Clause under Article I, Section 8 B The Police Power under Article I, Section 8 C The Taxing and Spending Power under Article I, Section 8 D The Privileges or Immunities Clause of the Fourteenth Amendment

Answer choice C is correct. The spending power has been interpreted very broadly. Congress has the power to spend for the "general welfare"—i.e., any public purpose—not just to pursue its other enumerated powers. Although there are areas in which Congress cannot directly regulate, it can use its spending power to accomplish such regulation indirectly by conditioning federal funding on certain behavior. Here, Congress conditioned federal grants on the adherence of grant recipients to standards for the treatment of laboratory animals. This is a proper use of the spending power for a public purpose. Answer choice A is incorrect. Congress is given the power to enact any legislation necessary and proper to execute any authority granted to any branch of the federal government. However, the Necessary and Proper Clause is not an independent source of power; it permits Congress's otherwise designated authority to be exercised fully. Answer choice B is incorrect because Congress does not have a specific police power to legislate for the public welfare. Such "police power" is reserved for the states. Congress can, however, tax and spend for the general welfare. Answer choice D is incorrect because the issue here is not one of infringement by the states upon the privileges or immunities of national citizenship. LEARN WHYNEXT QUESTION

A father, as he had been on three other occasions, was held in civil contempt for failure to pay child support payments to the mother of his child. The father was then incarcerated for six months. He appealed the court decision that found him in contempt. He contended that he was indigent and was entitled to the appointment of counsel to represent him at the contempt hearing. He argued that the court's failure to do so violated his rights under the Due Process Clause of the Fourteenth Amendment. By the time the father's appeal was heard, he had been released from prison. The state argues that although the father's financial and employment situation remained unchanged, the father's action should be dismissed as moot.How should the court rule on the state's defense? A For the state, because there was no live controversy after the father was released from prison. B For the state, because any further potential injury to the father would be speculative. C Against the state, because there is a reasonable expectation that the father will be subject to a civil contempt action again. D Against the state, because the father is entitled to counsel to represent him at a civil contempt hearing.

Answer choice C is correct. The state has defended against this action by arguing it is moot. Here, the father is no longer incarcerated and therefore no longer entitled to the appointment of counsel to represent him at the contempt hearing. However, since the father's financial and employment situation has not changed, there is a reasonable expectation that he will again be held in contempt for failure to pay child support, and may again be subject to incarceration without representation that terminates before his appeal could be heard. Consequently, under the mootness exception for an action that is capable of repetition yet evading review, the court should reject the state's defense. Accordingly, answer choice A is incorrect. Answer choice B is incorrect because the father, if indigent, has suffered harm by being incarcerated for six months without adequate representation by counsel. Not only is this harm capable of repetition yet evading review, a court's refusal to hear a matter because any potential injury would be speculative is an argument that the matter is not ripe for litigation, rather than that the action is moot. Answer choice D is incorrect because whether the father is entitled to the appointment of counsel is irrelevant to the question posed here, which is whether the court should recognize the state's defense that the action is moot.

A federal counterintelligence unit has been trying to locate a specific terrorist group for over a decade. A newspaper reaching an international audience gains knowledge of the unit's specific location and a plan for capturing members of the terrorist group. The plan for the capture of the terrorist group coincides with the imminent anniversary date of a prior terrorist attack by the group. Just prior to that date, the newspaper intends to publish an article that focuses on the prior attack, but in light of the plan, ends with the following words: "And to those who attacked our citizens: We know where you are, and we are coming to get you." The government files a motion for an injunction to prevent the newspaper from printing the story with that ending. The government asserts that the publication will alert the group to the impending attack and provide them with time to avoid capture. The government requests that the publisher not be allowed to publish the article as is until further notification from the counterintelligence unit. In responding to the motion, the publisher alleges that the article does not threaten national security, but fails to assert that the article is protected speech.Which of the following would the court least likely consider in its decision? A The words included in the article are insufficient to cause a particular harm. B The publisher was given a hearing before the injunction was issued. C The publisher cannot publish the article within the foreseeable future. D The publisher did not prove that the article was protected speech.

Answer choice D is correct. A prior restraint is a regulation of speech that occurs in advance of its expression (e.g., publication or utterance). In prior restraint cases, the burden is on the government to prove that the material to be censored is not protected speech. The publisher would therefore have no such burden, even if national security had been threatened. Answer choice A is incorrect because whether a particular harm might be avoided by prior restraint would be an important consideration. Answer choice B is incorrect because such a hearing would be evidence of a procedural safeguard highly relevant to determining whether a prior restraint is constitutional. Answer choice C is incorrect because a definite term for the restraint would be a safeguard available to the speaker. The fact that the restraint would occur "until further notification" is too indefinite and would be problematic for the censoring body.

Many state agricultural agencies conduct random discretionary inspections of meat processing facilities in their states. Relying only on its authority under the Commerce Clause, Congress passed a statute forbidding any state agricultural agency from considering whether a meat processing facility is licensed as "organic" when deciding when and where to conduct these inspections. The statute also provided that if a facility believed that the state agricultural agency was not following this statute in good faith, the facility could sue the state agency in federal court for damages. An organic meat processing facility in State A, believing that the State A agricultural agency conducted more random inspections of its facility than of non-organic facilities in the state, brought an action against the agency in federal court for damages. State A moved to dismiss the suit on the ground that it was immune under the Eleventh Amendment. Should the court dismiss the action on these grounds? A No, because the Eleventh Amendment only provides immunity for a state, not a state agency. B No, because Congress has unlimited authority to authorize private actions for damages against a state. C Yes, because a state may only be sued for damages if it consents to the suit. D Yes, because Congress cannot abrogate state immunity by exercising its Article I powers.

Answer choice D is correct. Although each of the Thirteenth, Fourteenth, and Fifteenth Amendments contains a provision that authorizes Congress to pass "appropriate legislation" to enforce the civil rights guaranteed by those amendments against the states, Congress cannot abrogate Eleventh Amendment immunity by exercising its Article I powers. Because this statute was based on the Commerce Clause of Article I, Section 8, Congress cannot constitutionally abrogate the state's Eleventh Amendment immunity. Answer choice A is incorrect because it is an incorrect statement of law. The Eleventh Amendment applies to states and state agencies. Answer choice B is incorrect because Congress does not have unlimited authority to authorize private actions for damages against a state. Congress cannot abrogate state immunity when exercising its Article I powers. Answer choice C is incorrect because it states only one exception to Eleventh Amendment immunity. If Congress had created this cause of action under the Thirteenth, Fourteenth, or Fifteenth Amendment, the state's lack of consent would not have preserved its sovereign immunity.

A state enacted a statute regulating a woman's ability to have an abortion. Among other pre-viability restrictions, the statute provides that a married woman who seeks to have an abortion must notify her spouse beforehand.Is the statute constitutional? A Yes, because it is rationally related to a legitimate state interest. B Yes, because it is necessary to further a compelling state interest. C No, because a woman's right to an abortion cannot be restricted pre-viability. D No, because it places an undue burden on a woman's right to an abortion.

Answer choice D is correct. An undue burden exists when the purpose or effect of a state law places substantial obstacles in the way of a woman's right to seek an abortion before the fetus attains viability. An undue burden has been found when a state requires a woman to notify her spouse before having an abortion, even when there are exceptions to the notice requirement. In this case, the statute requires a married woman to notify her spouse before having an abortion and indicates no exceptions. Therefore, the statute is almost certainly unconstitutional. Answer choice A is incorrect. A state restriction on abortion prior to viability must not, as noted with regard to answer choice D, place an undue burden on a woman's exercise of her right to an abortion, regardless of any compelling interest. Answer choice B is incorrect. In determining whether a restriction on a woman's right to have an abortion is constitutional, the undue burden test applies. Answer choice C is incorrect because a woman's right to an abortion may be restricted pre-viability, so long as the restrictions do not create an undue burden on a woman's right to seek an abortion. LEARN WHYFINISH

A state maintained its departments, including its Fish and Game Department, through tax revenues collected primarily from its residents. The department required all recreational deer hunters (i.e., those who hunt purely for sport) to obtain a deer hunting license. The license fee was $25 a year for state residents and $150 a year for out-of-state residents. An out-of-state resident wanted to go deer hunting for sport. He objected to paying a license fee that was six times the fee paid by in-state residents. He sued in an appropriate federal court. Will the court hold that the licensing fee scheme for recreational deer hunting is unconstitutional? A Yes, because the scheme violates the Equal Protection Clause by discriminating against out-of-state hunters. B Yes, because the scheme violates Article IV's Privileges and Immunities Clause by failing to accord out-of-state residents the same rights as state residents. C No, because the scheme is constitutionally valid under the Dormant Commerce Clause, as the subject of the fees, deer hunting, is a recreational activity, not a commercial one. D No, because the scheme is constitutionally valid under Article IV's Privileges and Immunities Clause, as recreational deer hunting is not a fundamental right, and a state may charge its residents a lower fee because their taxes support the Fish and Game Department.

Answer choice D is correct. Article IV's Privileges and Immunities Clause protects citizens of one state from discrimination by another state in their exercise of fundamental rights. Here, the state can charge higher licensing fees for out-of-state residents because recreational deer hunting is not a fundamental right, and the differential treatment is fair because state residents' taxes help to fund the state's Fish and Game Department. For this reason, answer choice B is incorrect. Answer choice A is incorrect because, since out-of-state deer hunters are not a suspect classification, the Equal Protection Clause merely requires that the state action satisfy the rational basis standard, which it does. Answer choice C is incorrect because the Dormant Commerce Clause applies only when a state discriminates against an out-of-state commercial actor, whereas here the out-of-state resident wishes to hunt for noncommercial, recreational purposes.

A federally owned national park is recognized by statute as a protected habitat for an endangered variety of edible mushroom. This type of mushroom is difficult to cultivate as a crop, but legal to purchase, own, and sell. However, because of the difficulty in growing the mushroom, there is a national black market of mushroom foragers specializing in stealing these mushrooms from public and private wilderness areas. To protect the endangered mushrooms in the national park, Congress passed a federal statute making it a criminal offense to pick the mushrooms on federal property.What is the strongest basis in support of the constitutionality of the federal statute? A The statute does not regulate a fundamental right. B The federal police power permits the protection of endangered wildlife. C There is a national market for the stolen mushrooms. D The statute is a necessary and proper means of protecting United States property.

Answer choice D is correct. Article IV, Section 3 gives Congress the power to make rules and regulate the territory and property belonging to the U.S. Here, the mushrooms are the property of the U.S., and can be properly regulated by federal law. Answer choice A is incorrect. The mere fact that the statute does not regulate or infringe upon an individual's fundamental rights does not provide a strong basis in support of the constitutionality of the statute. Answer choice B is incorrect because Congress has no general police power to legislate for the health, safety, welfare, or morals of citizens. Therefore, the protection of endangered wildlife could fall under no such power. Answer choice C is incorrect. Although this would have supported a regulation on the sale or ownership of the mushroom under the Commerce Clause, it is unnecessary for there to be a national market for these mushrooms for this statute to be constitutional. Because it only regulates theft from federal property, there is no need to apply the Commerce Clause to find that the statute is a valid exercise of congressional power.

After a prominent senator is accused of accepting bribes from commercial gaming lobbyists in exchange for a promise to vote in favor of land-based casinos in a state, a Congressional committee is assigned to investigate the alleged misconduct. Within the course of the investigation, the committee seeks to question Senator X, a fellow senator from the same state as the accused senator. Senator X is subpoenaed to appear before the Congressional committee without the presence of counsel to testify on whether he ever saw the accused senator accept money from a gaming lobbyist. Senator X fails to appear before the Congressional committee and is immediately cited for contempt.Is the citation of Senator X for contempt valid? A Yes, because Senator X did not appear before the committee despite receiving the subpoena. B Yes, because Congress has the right to investigate any matter within a legitimate legislative sphere. C No, because the committee exceeded its investigatory power. D No, because Senator X did not receive procedural due process.

Answer choice D is correct. Congress does not have an express power to investigate, but the Necessary and Proper clause allows Congress broad authority to conduct investigations incident to its power to legislate. While a subpoenaed witness who fails to appear before Congress or refuses to answer questions may be cited for contempt, the witness is entitled to certain procedural due process rights, including the presence of counsel. Here, because Senator X did not receive procedural due process, the citation for contempt would be invalidated. Answer choice A is incorrect because although Senator X could be cited for contempt if the other requirements (such as procedural due process and the privilege against self-incrimination) were met, the requirements of due process are not met here in light of the lack of counsel. Answer choice B is incorrect because while Congress can investigate all matters within a legitimate legislative sphere, which would include the possible acceptance of bribes in exchange for votes, any subpoenaed witness who fails to appear would still be entitled to procedural due process. Answer choice C is incorrect because the committee did not exceed its investigatory power, as whether the senator accepted bribes in exchange for votes related to commercial gaming would fall into the legislative sphere.

Investors brought an action under federal law for fraud in the sale of securities against an investment company. The action was dismissed with prejudice by the federal district court because it was not timely filed. The investors did not appeal this dismissal. Congress then passed legislation permitting the investors to reinstate this action. The investors petitioned the district court for reinstatement of their action.Of the following, which would serve as the best ground for the investment company to challenge the constitutionality of this law? A Bill of Attainder Clause of Article I, Section 9 of the U.S. Constitution B Due Process Clause of the Fourteenth Amendment C Ex Post Facto Clause of Article I, Section 9 of the U.S. Constitution D Separation of powers doctrine

Answer choice D is correct. Congress may not reinstate the right to bring a legal action after the judgment in the action has become final. Requiring a federal court to do so violates the separation of powers doctrine. Once a judicial decision becomes the final word of the federal judiciary with regard to a particular case or controversy, Congress may not declare by retroactive legislation that the law applicable to that particular case was different from what the courts said it was. Answer choice A is incorrect. A bill of attainder is a legislative act that declares a person or group of persons guilty of some crime and punishes them without a trial. Although the federal government as well as the states are constitutionally prohibited from enacting such legislation, the statute in question is not a bill of attainder because it did not declare the defendant investment company guilty of a crime and punish it without a trial. Answer choice B is incorrect because the Due Process Clause of the Fifth Amendment, not the Fourteenth Amendment, applies to the federal government. Answer choice C is incorrect. Although the constitutional prohibition on ex post facto laws does apply to the federal government as well as the states, the statute in question is not an ex post facto law because it did not constitute a retroactive change in a criminal or penal law (or a civil law with an overriding punitive effect). It merely attempted to permit the reinstatement of the investors' lawsuit.

A protester entered an IRS office during business hours. He denounced the income tax and set fire to pages from his copy of the Internal Revenue Code. The fire was extinguished before it caused any other damage. The protester was arrested and charged with violating a state law that prohibited igniting a fire in a public building. He claimed that his prosecution was unconstitutional under the First Amendment.May the protester constitutionally be convicted? A No, because he was exercising his right to freedom of speech by burning a copy of the code. B No, because the copy of the code belonged to him, and thus burning it did not infringe upon a legitimate government interest. C Yes, because the burning of the code was conduct rather than speech. D Yes, because the state law is narrowly drawn to further a substantial government interest in prohibiting the noncommunicative aspects of the act in question.

Answer choice D is correct. Expressive conduct (or symbolic speech) may be protected as speech, but it is subject to a lesser degree of protection. The protester's burning of the tax code qualifies as expressive conduct protected by the free speech clause of the First Amendment. Government regulation of expressive conduct is upheld if: i) The regulation is within the government's power to enact (e.g., through a local government's police power); ii) The regulation furthers an important government interest; iii) The government interest is unrelated to the suppression of ideas; and iv) The burden on speech is no greater than necessary. Because the state's interest underlying the law that the protester violated (preventing the burning of public buildings) is unrelated to the message communicated by the burning of the tax code, a court will not subject the state law to strict scrutiny. Instead, the court will uphold application of the law to the protester if the law is narrowly tailored to further a substantial government interest, a standard of justification that this law should satisfy easily. Answer choice A is incorrect because the court will be able to meet its burden here, especially as expressive conduct is subject to a lesser degree of protection. Answer choice B is incorrect because the legitimacy (and substantiality) of the government's interest in preventing the burning of public buildings is not undermined by the fact that the protester burned his own copy of the tax code. Answer choice C is incorrect because protected speech may include

A federal statute created a long-term program to fund a massive renovation and repair project for bridges in the interstate highway system. The statute provided that the Federal Highway Administration (FHA) "shall allocate and distribute the appropriated funds" in accordance with a specified statutory formula to state transportation agencies that agree to participate in the program. In the current fiscal year, Congress appropriated $80 million towards this program. However, because the President determined that the statutory allocation formula reflects the exaggerated influence of certain states on Congress rather than a prioritization of repair projects based solely on engineering concerns, the President directed the administrator of the FHA to first conduct a review of the bridges in the interstate highway system based solely on engineering safety criteria before allocating and distributing the funds on the basis of a formula keyed to those criteria. Since the study has not yet been completed, the President has directed the FHA administrator to delay the allocation and distribution of the appropriated funds until the next fiscal year.Assuming no other federal statutes are applicable, is the FHA administrator constitutionally required to allocate and distribute $80 million to state transportation agencies this year? A No, because it is the President's duty to execute the laws. B No, because the FHA adminstrator, as a member of the executive branch, is required to follow the President's directive. C Yes, because the President has no discretion whenever Congress has allocated funds to the states. D Yes, because the President may not unilaterally delay an explicitly mandated allocation and distribution of monies appropriated by Congress.

Answer choice D is correct. If Congress explicitly mandates an allocation, distribution, or expenditure of funds, the President lacks the power to impound those funds (e.g., refuse to spend them or delay their spending). Because Congress has mandated that these funds must be allocated and distributed in accordance with the statutory formula, the President cannot unilaterally impose a different allocation or distribution formula. Answer choice A is incorrect because the President, although required to execute the laws, must take care do so faithfully. Although the President is permitted to exercise spending discretion if the authorizing legislation so provides, that is not the case here. The President's power does not extend to delaying the allocation of funds that Congress has mandated to be allocated or to allocating the funds through a different formula than the statutorily provided formula. Answer choice B is incorrect because, although the FHA administrator is subject to the direction of the President, the President here lacks the authority to require the FHA administrator to ignore the Congressional mandate with regard to allocation and distribution of authorized funds. Answer choice C is incorrect because the President is not always prohibited from exercising discretion when Congress allocates funds to the states, but only when Congress mandates the funds be allocated to the states and provides a formula for doing so. LEARN WHYNEXT QUESTION

A state law authorized slot machines at various locations in the state and imposed a tax on revenues from the slot machines at a rate of 35 percent. In response to the financial peril in which the state riverboat industry found itself, the state lowered the tax rate on revenues from slot machines found on riverboats in the state, which represented a substantial portion of the riverboat industry's income, to 20 percent. The owners of the venues paying the 35 percent tax rate on their slot machine revenue filed an action in federal court challenging the constitutionality of the different tax rates depending upon the type of venue because it violates the Equal Protection Clause.Is the court likely to rule in favor of the plaintiffs? A Yes, because the state was favoring one intrastate venue over other types of venues. B Yes, because the law violated the geographic uniformity requirement imposed by Article I, Section 8. C No, because the subsidy exception of the Dormant Commerce Clause applies. D No, because the different tax rates were rationally related to a legitimate state interest.

Answer choice D is correct. Since the plaintiffs have challenged the differential tax rates based on equal protection grounds, and because the plaintiffs are not members of a suspect or quasi-suspect class, the discriminatory tax rate is only subject to the rational basis test. Here, the state acted to aid a state industry (the riverboat industry) that was in financial peril (a legitimate state interest) by lowering the tax rate on one form of revenue that generated a substantial portion of its income. While the state could have undertaken other actions to aid the riverboat industry, the modification of the tax rate on slot machine revenue was rationally related to the state's interest in reducing the riverboat industry's financial peril. Consequently, the court is likely to rule against the venue owners paying the 35 percent tax rate. Answer choice A is incorrect because, unlike state action that discriminates against interstate commerce, there is no specific constitutional restriction on state action that discriminates against intrastate business in favor of another intrastate business. Moreover, the plaintiffs' challenge is not based on the Dormant Commerce Clause, but on the Equal Protection Clause. Answer choice B is incorrect because the geographic uniformity requirement is a constitutional restriction imposed on the federal government, not on the states. Moreover, as stated above, the plaintiffs' challenge is based on the Equal Protection Clause. Answer choice C is incorrect because, while a state may provide a subsidy for its own citizens under the Dormant Commerce Clause (e.g., a lower tuition rate for in-state students attending college), the plaintiffs' challenge is based on the Equal Protection Clause.

A federal statute, applicable to all public and private employers, makes discrimination against persons with a certain disability in the hiring or firing of employees illegal and provides for a civil action for damages against the offending employer. The statute contains a clear statement of Congressional intent to abrogate state immunity.An employee sued her employer in federal court for violation of this statute and sought relief in the form of retroactive money damages. The employer, a state agency, moved to dismiss the action as constitutionally prohibited. The employee conceded that the state agency's acts, like most historical acts of discrimination against individuals with this particular disability, were rationally related to a legitimate government purpose.Should the court dismiss the action on constitutional grounds? A No, because Congress was acting pursuant to its power under the Enabling Clause of the Fourteenth Amendment. B No, because Congress was acting pursuant to its power under the Commerce Clause. C Yes, because the state agency's actions did not violate the Equal Protection Clause. D Yes, because the Eleventh Amendment prevents the recovery of retroactive money damages by citizens against a state agency.

Answer choice D is correct. The Court has interpreted the Eleventh Amendment as barring unconsented private suits against a state for retroactive money damages. Congress may expressly abrogate state immunity only if it is clearly acting under Section Five of the Fourteenth Amendment to enforce rights created by Section One (or to enforce rights created by the remedial provisions of the other Civil War Amendments. Here, Congress is not remedying conduct that violates the Fourteenth Amendment's Equal Protection Clause, as disability is not a suspect or quasi-suspect class and thus a state's disability-based discrimination merely needs a rational basis. That is why answer choice A is incorrect. Answer choice B is incorrect because Congress cannot abrogate state sovereign immunity under the Commerce Clause. Answer choice C is incorrect because, since the actions of the state agency (and similar actions by other states) did not violate the Equal Protection Clause in Section 1 of the Fourteenth Amendment, Congress has no remedial power under Section Five of that Amendment to enact this statute at all, much less to abrogate state sovereign immunity. LEARN WHYNEXT QUESTION

Congress has enacted many laws regulating navigation generally, but not regarding the specific subject of water pollution by ships sailing on navigable bodies of water. A state enacted a law prohibiting any ship from discharging specified pollutants, including oil, into the navigable waterways of the state. Violation of the law was punishable by fines based on the amount of the discharge. The law is necessary to the important state interest of preventing pollution; there are no reasonable alternatives available. In addition, the benefits of the law to the state outweigh the burdens it imposes on interstate commerce. A ship owner from another state is fined pursuant to this law for discharging oil into a waterway in the state. Will the ship owner's challenge to the state law as unconstitutional be successful? A Yes, because the law regulates interstate commerce, which may be regulated only by Congress. B Yes, because the fine constitutes an impermissible ad valorem tax. C No, because the law is necessary to the important state interest of preventing pollution and there are no reasonable alternatives available. D No, because the law does not discriminate against interstate commerce and does not impose an undue burden on interstate commerce.

Answer choice D is correct. The Dormant Commerce Clause requires that a state law not discriminate against out-of-state commerce and not constitute an undue burden on interstate commerce. The law in question satisfies these two requirements. Answer choice A is incorrect because although Congress has the power to regulate interstate commerce, if Congress has not acted with respect to a particular aspect of interstate commerce, a state may regulate that aspect. Answer choice B is incorrect because the law imposes a fine, not a tax. Even if the fine were construed to constitute a tax, it is not assessed on the value of property, and therefore is not an ad valorem tax. Answer choice C is incorrect because, since the law does not discriminate against out-of-state commerce, it is irrelevant that the law satisfies the exception that permits discriminatory laws if they are necessary to an important state interest.

The Occupational Safety and Health Act of 1970 (OSHA) required all private employers in America to meet certain minimum federal standards to ensure safe and healthful work environments. Recently, Congress amended OSHA, extending its coverage requirements to state and local government employers. A state sued in an appropriate federal court, challenging the constitutionality of this amendment. Will the court likely uphold the amendment as constitutional? A No, because the amendment violates the Tenth Amendment. B No, because the amendment violates fundamental principles of federalism, because Congress has directly impaired the states' ability to carry out their integral governmental functions. C Yes, because the amendment merely affects the activities of states acting in their proprietary capacity. D Yes, because the amendment is a valid exercise of Congress's Commerce Clause power.

Answer choice D is correct. The amendment regulates an intrastate economic activity (i.e., the workplace itself) that has a substantial effect on interstate commerce. Answer choices A and B are incorrect because neither the Tenth Amendment nor the principle of federalism prevents the application of the federal standards to the state workers. Answer choice C is incorrect because Congress has plenary Commerce Clause power to regulate employment conditions of all state employees.

A city adopted an ordinance that prohibited any apartment building from housing more than one convicted felon on the premises. A convicted felon owned an apartment building in the city. He lived in one of the apartments in the building with his brother, who was also a convicted felon. The building owner received a citation from the city for violating the ordinance, and was ordered to pay a fine and bring the building into compliance. The building owner sued the city, arguing that the ordinance violated his constitutional rights.What is the best argument for striking down the ordinance? A The ordinance violates the Privileges and Immunities Clause. B The ordinance violates the Equal Protection Clause. C The ordinance violates the Eighth Amendment. D The ordinance violates the Due Process Clause.

Answer choice D is correct. The guarantee of substantive due process is based upon the idea that laws should be reasonable and not arbitrary, and ensures that a governmental action that infringes upon a fundamental right is generally subject to strict scrutiny. Related persons have a fundamental right to live together in a single household. In this case, the ordinance prevented the man from living with his brother. Thus, the man's best argument is that the ordinance, by prohibiting him from living with a family member, violates substantive due process. Answer choice A is incorrect because the Privileges and Immunities Clause, which prohibits a state from discriminating against the citizens of another state, is not implicated in this case. Answer choice B is incorrect because the law does not employ a suspect classification, and thus any equal protection claim would be subject to rational basis review. The man would have a difficult time showing that the ordinance did not satisfy this standard. Answer choice C is incorrect because state and local governments do not violate the Eighth Amendment simply by imposing special disabilities on convicted felons, especially when doing so is reasonably related to protecting public safety.

A state amended its constitution to provide that "English is the state's official language and applies to all state employees during the performance of government business." A state employee sued the state's governor to enjoin application of this provision to her. She alleged that she worked in the state's Department of Motor Vehicles and often communicated with customers in Spanish, thereby facing possible adverse employment action in violation of her First and Fourteenth Amendment rights. The governor defends against this action by invoking the political question doctrine. Will the governor likely be successful? A Yes, because the case presents a hotly contested question involving sensitive political issues. B Yes, because a judicial decision would unnecessarily embarrass the governor. C No, because the law does not require the judge to make any discretionary policy determinations. D No, because the state constitutional provision has been properly challenged as violating individual constitutional rights.

Answer choice D is correct. The political question doctrine applies only to the federal government, not the states. Here, the state employee has brought a plausible claim that her individual constitutional rights have been violated. Consequently, answer choices B and C are incorrect because they are irrelevant to this case. Although they both set forth one of the factors used to determine whether a question is "political," neither applies because a state law is involved. Answer choice A is incorrect because the political question doctrine focuses on whether the Constitution has assigned decision-making on this subject to a different branch of government or if the matter is inherently one that the judiciary cannot decide. The political question doctrine does not necessarily preclude a court from deciding a matter that involves sensitive political issues or presents a hotly contested question.

Congress enacted a statute establishing a program to protect areas in the United States that are rich in biological diversity. The program is consistent with the terms of an environmental treaty that the President objected to and did not sign. The statute creates an executive agency and authorizes it to designate parts of federal lands for inclusion in the program in accordance with criteria taken from the treaty. In an inseverable provision, the statute further provides that the agency must report each designation to a committee of Congress and that the committee may overturn the agency's designation by a majority vote.Why is the statute unconstitutional? A It constitutes an invalid delegation of legislative authority to an executive agency. B It interferes with the exercise of the President's paramount authority in foreign affairs. C It requires an executive agency to report its decisions to Congress. D It authorizes a committee of Congress to overturn an executive decision.

Answer choice D is correct. The provision of the statute authorizing a congressional committee to overturn the agency's designations of federal lands is an unconstitutional legislative veto. Congress may overturn the action of an executive agency only by enacting a statute. Answer choice A is incorrect because the statute's grant of authority to the agency does not violate the non-delegation doctrine because the statute's incorporation of the treaty criteria provides intelligible principles sufficient to guide the agency's designations of federal lands. Answer choice B is incorrect because Congress has power to legislate regarding the control of federal lands, and the statute does not affect the President's conduct of foreign affairs. Answer choice C is incorrect because the necessary and proper clause authorizes Congress to require that executive agencies report their decisions to Congress if such a requirement reasonably implements a statutory program.

A public co-ed high school has adopted a requirement that boys who play any school-sponsored sport cannot have hair longer than four inches. The school does not impose a similar requirement on girls who play a school-sponsored sport. A male student in the school challenges this requirement in federal court on the grounds that it interferes with his right to wear his hair as he pleases.Which of the following is likely the best statement of the burden of persuasion for the action brought by the student? A The school is required to establish that the school requirement is the least restrictive means to achieve a compelling interest of the school. B The school is required to establish that the requirement is substantially related to an important interest of the school. C The student is required to establish that the requirement is not substantially related to an important interest of the school. D The student is required to establish that the school requirement is arbitrary or irrational.

Answer choice D is correct. The standard of review to which governmental action is subjected when challenged on substantive due process grounds depends on the right asserted by the plaintiff. If the plaintiff is asserting a fundamental right, the government must establish that its requirement is the least restrictive means to achieve a compelling governmental interest. If a fundamental right is not involved, the plaintiff must show that the restriction does not bear a rational relationship to a legitimate government interest. Laws are presumed valid under this standard, so the burden is on the challenger to overcome this presumption by establishing that the law is arbitrary or irrational. Here, the student's interest in being able to wear his hair as he chooses is merely a life style choice that does not involve a fundamental right. Consequently, the burden is on the student to show that the requirement is arbitrary or irrational. Answer choice A is incorrect. In addition to governmental conduct that violates a fundamental right, the strict scrutiny standard set out in this answer choice also applies to intentional government conduct that discriminates against a suspect class. Because the facts do not indicate that the student belongs to a suspect class and a fundamental right is not involved, strict scrutiny review of the school hair requirement is not appropriate. Answer choice B is incorrect. The intermediate scrutiny standard of review set out in this answer choice is appropriate for equal protection challenges when intentional government conduct discriminates against a quasi-suspect class. Although the student here could have pursued a constitutional challenge based on equal protection grounds (i.e., gender discrimination), he did not, and instead has brought a substantive due process claim based on the denial of a right to wear his hair as he wished. Consequently, this standard of review does not apply to his substantive due process challenge. Answer choice C is incorrect because the intermediate scrutiny standard of review does not apply to a substantive due process challenge. In addition, even if the action had been brought on equal protection grounds, the burden would be on the school, not the student. LEARN WHYNEXT QUESTION

A city airport is one of the busiest in the nation and caters to travelers of all kinds, particularly sports stars. The airport had struggled for many years to maintain order at security checkpoints when these sports stars were recognized by fans. For security reasons, the board of the city's airport commission used government funds to build a special terminal with a separate, faster security checkpoint where sports stars could go without having to interact with the general public. A recent study that was given to the board showed that female sports stars earn an average of $150,000 per year, whereas male sports stars earn an average of $400,000 per year. The same study suggested that although there is a correlation between an athlete's income and his or her popularity among fans, the two factors are not directly proportional. To prevent the need for a costly and time-consuming application process for athletes wanting to use the special terminal, the board required that a sports star must earn a yearly salary of at least $300,000 to use the private terminal. A female sports star who earns $175,000 per year would like to use the special terminal, but is not permitted to do so. The female sports star sued the board, claiming that the minimum salary requirement violates her constitutional rights. Which of the following argument's best supports the female sports star's challenge to the minimum salary requirement? A The requirement is not rationally related to the likelihood of a sports star's presence causing a security disruption. B The requirement is discriminatorily applied to female sports stars. C The requirement has a disparate effect on female sports stars. D The board's prior knowledge of the study evidences a discriminatory intent in setting the minimum salary requirement.

Answer choice D is correct. To trigger strict or intermediate scrutiny under the Equal Protection Clause of the Fourteenth Amendment, there must be discriminatory intent on the part of the government. The fact that legislation has a disparate effect on people of different races, genders, etc., without intent, is not sufficient to trigger strict or intermediate scrutiny. Instead, proof of discriminatory motive or intent is required. Here, the female sports star should argue that there is discriminatory intent because the board was aware of the fact that the average female star earns far less than $300,000. Thus, there would be a disparate impact on female sports stars coupled with a discriminatory intent, thereby triggering intermediate scrutiny, as gender is a quasi-suspect class. Answer choice A is incorrect. Even though the relationship between a sports star's salary and his or her popularity is not directly proportional, there was a correlation shown by the study given to the board. Therefore, if the female sports star makes an argument applying rational basis, her challenge is likely to fail, as laws are presumed valid under this standard. Answer choice B is incorrect. A law that appears neutral on its face may be applied in a discriminatory fashion. However, there is no evidence that the requirement in question is not applied identically to all sports stars, male and female. Therefore, this is not the female sports star's strongest argument. Answer choice C is incorrect because the fact that legislation has a disparate effect on people of different races, genders, etc., without intent, is not sufficient to trigger strict or intermediate scrutiny.

A state recently enacted a controversial statute regarding a presumptive father's rights concerning an abortion. The state judiciary has not yet determined the constitutionality of the statute. The statute has resulted in such public outrage that violent demonstrations are erupting around all of the abortion clinics in the state and outside the state legislative building. Leaders of the opposition to the statute have vowed to continue to act until the statute is repealed or overturned. Congress has authorized the President to call National Guard units into the state to guard the state legislative building, but not the abortion clinics. The abortion clinics in the state have brought a federal suit seeking protection from the National Guard. The clinics have not yet been damaged in any way.Which of the following arguments would best support the dismissal of this suit? A The suit will be moot by the time a verdict is rendered. B The suit is not ripe for litigation because no injury has occurred yet. C The suit involves an unsettled question of state law. D The suit involves a nonjusticiable political question.

Answer choice D is correct. Under certain circumstances, the courts will refuse to hear a case or issue an opinion. In particular, a federal court will not rule on a matter in controversy if it is a political question, meaning (i) the Constitution has assigned final decision-making on this subject to a different branch of government, or (ii) the matter is inherently not one the judiciary can decide. The ability of Congress to authorize the President to call the National Guard and the President's choice to do so are political questions on which the federal court should not rule. Answer choice A is incorrect because, based on the vow of the leaders of the opposition, it is unlikely that the violent demonstrations will have ended by the time a verdict is rendered. Additionally, even if the violent demonstrations are so brief as to evade review, they may occur again, putting the clinics at risk. Therefore, this is not the strongest argument for dismissal of the suit. Answer choice B is incorrect because an imminent threat of an injury can be enough to make a case ripe for litigation. Answer choice C is incorrect because, although the state constitutionality of the state statute has not yet been decided, the challenge brought by the clinic is unrelated to this unsettled issue of state law. LEARN WHYNEXT QUESTION

A state law provides that the Registrar of Motor Vehicles, upon receipt of a police report that a driver who was arrested for operating a motor vehicle under the influence refused to submit to a breath-analysis test, must immediately suspend the driver's license for 90 days. The driver, upon surrendering his license, is then entitled to a "same day" hearing before the Registrar.Of the following, which is NOT a factor in determining whether this failure to provide a pre-suspension hearing violates the Due Process Clause of the Fourteenth Amendment? A The degree to which reliability would be materially enhanced by mandating a pre-suspension hearing B The driver's interest in the continued possession and use of his license pending the outcome of the hearing C The risk of erroneous observation or deliberate misrepresentation by the reporting police officer of the facts forming the basis of the suspension D Whether the state can demonstrate a compelling interest in not providing a pre-suspension hearing

Answer choice D is correct. When an individual's protected interest is threatened by governmental action, a court considers three factors in determining the amount of process that is due: (i) the private interest affected by the governmental action; (ii) the risk of erroneous deprivation of that interest using current procedures and the probable value of additional or substitute safeguards; and (iii) the burden involved in providing the additional process. A state has the burden to demonstrate a compelling interest justifying its actions when a fundamental right is at issue. Answer choice A is incorrect because the degree to which reliability would be materially enhanced by mandating a pre-suspension hearing focuses on the probable value of a pre-suspension hearing. Answer choice B is incorrect because the driver's interest in the continued possession and use of his license pending the outcome of the hearing is the driver's interest that is affected by the immediate license suspension. Answer choice C is incorrect because the risk of erroneous observation or deliberate misrepresentation by the reporting police officer of the facts forming the basis of the suspension reflects the risk of erroneous deprivation of the driver's interest affected by the license suspension. LEARN WHYNEXT QUESTION

A long-standing federal law provides for the payment of a $250,000 death benefit to the immediate family of a firefighter who dies as a direct result of fighting a fire. The law provides that this payment is in addition to any other benefit paid to the firefighter's family from another source, but does not otherwise address the relationship between this law and any conflicting state law. Acting pursuant to a recently enacted state law, a state worker's compensation commission reduced the amount of compensation paid to the widow of a firefighter by the amount of the federal death benefit. The widow challenged the constitutionality of the state law.How should the court rule on the challenge? A Deny the challenge, because the commission's action did not affect the federal death benefit received by the firefighter's widow. B Deny the challenge, because the federal law does not state that it expressly preempts conflicting state laws. C Uphold the challenge, because the new state law was enacted after the long-standing federal law. D Uphold the challenge, because the state law violates the Supremacy Clause of the United States Constitution.

Answer choice D is correct. While a federal law that expressly prohibits conflicting state laws is enforceable based upon the Supremacy Clause, the Supremacy Clause has also been interpreted as providing for implied preemption of a state law by a federal law when the two directly conflict. Here, the federal law specifies that the federal benefit is in addition to any benefits from another source, while the state law reduces the amount of the state worker's compensation payment by the amount of the federal benefit. For this reason, federal preemption is implied, and answer choice B is incorrect. Answer choice A is incorrect because, while the state law did not result in the direct reduction of the federal death benefit paid to the widow, the state law did conflict with the language in the federal statute stating that the federal death benefit was in addition to any other benefit from another source. Answer choice C is incorrect because federal preemption is not limited to conflicting state laws enacted after the passage of a federal law. Under the Supremacy Clause, any state law that directly conflicts with a federal law, even those state laws that predate the federal law, are preempted by the federal law.

To address a shortfall in the judiciary's budget and to comply with a provision in the state constitution requiring a balanced budget, a state increased, by statute, various court fees, including the fee to file an appeal, and eliminated any waiver of these fees except for a defendant convicted of a capital crime and sentenced to death. A defendant imprisoned after his conviction of a non-capital offense filed an appeal and a request for a waiver of the fee because of his inability to pay. Citing the statute, the state appellate court ruled that the defendant was not entitled to the waiver, as he was not convicted of a capital offense and sentenced to death.The defendant properly challenged this ruling in federal court. Should the federal court uphold the state appellate court's ruling? A Yes, because the fee is rationally related to the goal of adequately funding the state's judicial system. B Yes, because state law required payment of the fee. C No, because a state may not prohibit a prisoner from exercising a fundamental right. D No, because the defendant was indigent.

Answer choice D is correct. While most state actions that discriminate against the poor are subject only to rational basis scrutiny, the availability of appeal in a criminal case cannot hinge on ability to pay a filing fee. To deny an indigent the right to appeal through the imposition of a fee violates the Due Process and Equal Protection Clauses of the Fourteenth Amendment. Answer choice A is incorrect because, although there is a rational basis for the state's increase in the various court fees, an indigent accused has a constitutional right to waiver of a fee that prevents him from enjoying a fundamental right, such as access to the courts. Answer choice B is incorrect because, even though the state court decision was based on state law, the state court cannot enforce a state law that conflicts with the federal constitution. Answer choice C is incorrect because, while a prisoner has a right of reasonable access to the court, the law in question did not deny the defendant such access on the basis of his status as a prisoner, but due to his failure to pay the necessary filing fee.

what's the president privilege?

Power to withhold official documents & communications: National security secrets - afforded greatest deference Civil case - afforded greater deference than in criminal trial Criminal case - afforded least deference (disclosure required if prosecution shows need)

what type of classification does the strict scrutiny apply?

o 1.The classification is "suspect"—that is, it's based on race, ethnicity or alienage (non-U.S. citizenship); OR o 2.The classification relates to who may exercise a fundamental right § Freedom of association, § Interstate travel - look for residency requirements as conditions to receipt of welfare benefits, medical care, or voting rights, which are considered vital governmental services only, not included tuition reduction or the right to a divorce; § Privacy (marriage, procreation, child-rearing, abortion) § Voting (requirements other than age, residency, and citizenship) § If alienage (i.e., non-citizenship) is the basis of the classification: Strict scrutiny is not given when a state discriminates against non-citizens in the context of jobs performing essential state functions. Thus, a state is subject only to rational-relation review if it requires that state police, public school teachers or probation officers be U.S. citizens.

· Commerce clause: 3 main areas of interstate Commerce that Congress can regulate:

o Instrumentalities (trains, planes, and other vehicles) o Channels (roads, railroads, airways, and navigable waterways) o Economic activity that if aggregated across the national economy substantially affects interstate commerce § Can regulate grow of marijuana for personal use because in the aggregate it will affect economic activity across the national economy

what are the elements for standing?

o To have standing, a plaintiff bears the burden of establishing three elements: (i) there is an injury in fact, (ii) the injury must be caused by the defendant's violation of a constitutional or other federal right, and (iii) the relief requested must prevent or redress the injury. While the threat of future injury can suffice, it cannot be merely hypothetical or conjectural, but must be actual and imminent.

what is the scrutiny used for state laws that discriminate against aliens?

strict scrutiny o the "compelling interest" test: the classification scheme must be necessary (The least restrictive means) to achieve a compelling governmental interest. (For federal law, alienage classification is likely valid unless it is arbitrary and unreasonable; If State law related to alien's participation in government business, then it is rational relation);

how can speech in public forum be regulated?

§ Only time, place, and manner may be regulated in a public forum; the restrictions must: 1. be content-neutral as to both subject matter and viewpoint, 2. be narrowly tailored to serve a significant governmental interest, and 3. leave open ample alternative channels for communication. § Apply this test when a group needs to get a permit or license before that group can speak or demonstrate. Note that the ordinance or regulation must set out the grounds for denying a permit in narrow and specific language that curtails the discretion of local officials. (If it doesn't, the ordinance or regulation is probably overbroad.)

o Privileges & Immunities of the Fourteenth Amendment.

§ This voids state enactments which clearly infringe on the privileges of national citizenship. The protection is limited to the fundamental rights shared by all citizens, namely the right to travel freely from state to state, to petition Congress for redress of grievances, to vote for national officers, to assemble peaceably, and to discuss matters of national legislation. IT IS USUALLY A WRONG ANSWER: wherever Privileges and Immunities seems to apply, Due Process or Equal Protection would be a stronger argument against the constitutionality of the state action in question


Conjuntos de estudio relacionados

CE BOARD EXAM TERMINOLOGIES - HIGHWAY ENGIINEERING

View Set

Abeka 7th grade Literature test 9

View Set